Download MODULE 1 dental № TOPIC TEST 1. Histology as object

Document related concepts

Nerve guidance conduit wikipedia , lookup

Auditory system wikipedia , lookup

Cell encapsulation wikipedia , lookup

Bio-MEMS wikipedia , lookup

Organ-on-a-chip wikipedia , lookup

Tissue engineering wikipedia , lookup

Transcript
MODULE 1 dental
№
TOPIC
1. Histology as
object.
Microscope.
Research methods
are in histology
Methods of
making of
histological
preparations for a
light and
electronic
microscopy.
2. Basic
organization of
cell. Cellular
surface
3.
Structure of
cytoplasm.
Organelles and
inclusions
TEST
On the electrocardiogram of the man of 23 years there are attributes of infringement of
carrying out of excitation from auricles to ventricules (that is connected to
infringement of an exchange of ions between cells), caused rheumatic myocarditis.
What change of structures of contacting surfaces of cells of heart these phenomena
most authentically speak?
A. Gap junctions*
B. Microvillies
C. Tight junctions
D. Desmosome
E. Nexuses
At research of blood at the patient increase of lactatdehydrogenase’s activity is
revealed. What changes in an organism at a cellular level result in similar
infringements?
A.Destruction of plasmatic membranes *
В.Metabolic disfunction
С. Infringement of intercellular interactions
D. Damage of the genom
E.Decreased activity of enzymes’ system
A 50 year old woman tooth extracted. The tissue regenerated. Which of the following
organella are the most active during tissue regeneration?
A Ribosomes*
B Centrosomes
C Postlysosomes
D Agranular endoplasmic reticulum
E Lysosomes
Formation of ribosome subunits in a cell was disturbed in course of an experiment (by
means of activated mutagenic factors). This will have an effect on the following
metabolic process:
A Protein biosynthesis*
B Carbohydrate biosynthesis
C ATP synthesis
D Photosynthesis
E Biological oxidation
Electron microscopic study of a cell revealed roundish bubbles confined by a
membrane and containing a lot of various hydrolytic enzymes. It is known that these
organellas provide intracellular digestion and protective functions. These elements
are:
A Lysosomes*
B Centrosomes
C Endoplasmic reticulum
D Ribosomes
E Mitochondria
1
An animal had been intensively fed with carbohydrates. Histologic examination of its
liver revealed a significant number of glycogen granules. Glycogen relates to the
following group of cell structures:
A Trophic granules*
B Secretory granules
C Excretory granules
D Pigment granules
E Special organelles
At the man 42 years old for specification of the diagnosis it is lead is carried out biopsy
of the liver. At research of biopsy material it was revealed, that its cells have risen
increased of cytoplasms’ basophylia. It testifies that in cells occurs:
A. Activation of protein syntesis*
B. Formation of lipids
C. Activation of transmembranouse transport
D. Active absorbtion of substances
E. Mitotic division
Cells of a laboratory animal to process by superfluous x-ray radiation. In result
protein’s fragments in cytoplasm were formed. What organelles of cells will take
participation in their recycling?
A.Lуsosomes*
B. Golgy complex
C. Ribosomes
D. Endoplasmic reticulum
E.Cell center
Mucopolysacharidosis concerns to illnesses of accumulation. For the lack of enzymes
that destroing polysacharides is broken. At patients increase of their allocation with
urine and accumulation in one of cells’ organelles is observed. In what organelles
collect mucopolysacharides?
А. In Golgy complex
В. In Lisosomes
С. In cellular center
D. In endoplasmic reticulum
Е. In mitochondrias
For the person it is diagnosed galactozemia - illness of accumulation. As a result of
infringement of what cellular structure there was this illness?
A.Golgy complex
B.Centrioles
C.Lisosomes*
D. Cellular center
E.Mitochondrias
At electron micrographs of nerve cells of spinal node there were found organelles.
They were consisted of tanks flattened in the central and extended to the periphery
and small vesicles. What are these organelles?
A Lysosomes
B Centrioles
C Golgi complex*
D peroxisomes
E Mitochondria
During histochemical research of hepatocyte in the cytoplasm the were detected
vesicles diameter 0,05-1,5 micron with peroxidation enzymes - catalase, peroxidase.
What are these organelles?
A. Lysosomes
B. Peroxisomes*
2
C Melanosomes.
D Liposomes.
E Phagosome.
Low level of albumins and fibrinogen was detected in the patient's blood. Decreased
activity of what organelle of the liver hepatocytes can cause it?
A Granular endoplasmatic reeticulum*
B Agranular endoplasmatic reticulum
C Mitochondrions
D Golgi complex
E Lysosomes
Аt ultramicroscopic research of hepatocytes in there cytoplasm rough endoplasmic
reticulum was determined. What is the function of this organell?
A Synthesis of plasma proteins*
B Synthesis of carbohydrates
C detoxification.
D production of bile.
E depositing calcium ions.
The tumour of a uterus is removed the woman of 67 years. At histologic research in
cells of a tumour are found out multipolar mitosises - pictures of a divergence to
several poles. With infringement of what organelles’ condition the most authentic
occurrence multipolar mitosises?
A. Secondary lysosomes
B. Smooth endoplasmic reticulum
C. Rough endoplasmic reticulum
D. Peroxysomes
E. Centrioles*
In a nutrient substance where cells of animals are raised, have added aminoacid leucin
with a radioactive label. After a while a method of radioautography have found out
high concentration labeled aminoacid near certain determined organelles. These
organelles can be:
A. Ribosomes*
B. Lysosomes
C. Smooth endoplasmic reticulum
D. The cellular center
E. Golgy complex
In the cytoplasm of pancreatic cells during the secretory cycle secretory granules are
appearing and disappearing in apical part of cell. Which structural elements can be
attributed these granules?
A To inclusions*
B To microfilaments
C To lysosomes
D To Golgy complex
E To rough endoplasmic reticulum
4.
Cell nucleus
Mitosis. Cellular
cycle.
In a nutrient substance with cells which are cultivated, the solution of Thimin (Т) with
a radioactive label is brought. In what structures of cells will find out designated
Thimin during radioautography?
A. Nucleus*
B. Ribosomes
C. Golgy complex
D. Endoplasmic reticulum
E. Lysosomes
In culture of tissue by a nuclear irradiation are damaged injured nucleoles. What
3
organelles renewaled in cytoplasm of cells become problematic?
A. Smooth endoplasmic reticulum
B.Ribosomes*
C.Lysosomes
D. Golgy complex
E.Microtubules
In the cell the synthesis of histone proteins was blocked artificially. What cell
structure is damaged?
A Golgi complex
B Nucleolus
C Nuclear chromatin*
D Cell membrane
E Nuclear membrane
During division of a cell the researcher managed to observe a phase at which were
absent a membrane of a nucleus and nucleola, and centrioles were on poles of a cell.
Chromosomes looked like a ball of strings which are freely located in cytoplasm. It is
typical of what phase?
A.Metaphase
B.Prophase*
C.Telophase
D. Anaphase
E.Interphase
Doctor made smea from mucous shall of the oral cavity of the person. In not destroyed
epithelial cells painted smea it is well visible oval nucleus, unequal in the sizes. What
way occured division of these cells?
A.Mitosis
B.Endomitosis
C.Shizogonia
D. Amitotic division *
E.Mejosis
At microscopic research of liver’s tissue have been found out, that some cells have
broken up to small fragments with separate organelles and nucleus’ fragments
surrounded with a membranes. Inflammatory reaction is absent. For what
pathological process these changes are characteristic?
A.Apoptosis *
B.Karyoreksis
C.Plasmoreksis
D. Plasmolisis
E.Nekrosis
5.
6.
Bases of general
embryology and
gametogenesis.
Fertilization.
Implantation.
Formation of
extraembryonic
shells.
At a certain stage of development of a human embryo one can observe formation of a
cavity in its structure, small light blastomeres on the periphery and large dark
blastomeres at one of the poles. The embryo at this stage of development is called:
A Blastocyst*
B Morula
C Zygote
D Gastrula
E Blastodisk
For an unknown reason the fertilization membrane of an embryo dissolved in the
fFllopian tube in the first critical period. What complication of pregnancy is possible
in this case?
4
A Embryo implantation into the Fallopian tube*
B Embryonic death
C Invagination of the blastocyst wall
D Return of blastocyst back to the ampullary portion of the tube
E Formation of two blastocysts
Implantation process has two stages: adhesion and invasion. Morphological
manifestation of blastocyte adhesion is:
A Attachment of blastocyte to the endometrium*
B Destruction of endometrium epithelium
C Destruction of connective tissue of endometrium
D Destruction of endometrium vessels
E Formation of lacunes
During experiment above frog’s blastula at a stage of 16 blastomeres have been
removed 1 blastomere. This cell continued to develop normally and has begun a new
germ. What is important property of blastomeres was is shown?
A.Totipotentnist*
B.Embryionic induction
C.Differentiation
D. Formation of poles of an embryo
E.Germinal layers’ formation
At microscopic research of embryo bodies which are removed during operation, the
germ consists of two blastomeres has been found. Named a place of its localization in
condition of normal embryo’s development.
A. Ovaries
B. Uterine tube near uterus
C. Uterine cavity
D. Ampullary part of the uterine tube *
E. Abdominal cavity
Process of segmentation of zygote comes to the end with formation of blastocyst.
What is the type of human blastocyst?
A. Morula
B. Celoblastula
C. Discoblastula
D. Amphyblastula
E. Blastocyst *
7.
Gastrulation.
Tissue- and
organogenesis.
Human embryo was found in the uterus. He wasn’t attached to the endometrium.
What stage of development the embryo is responsible?
A Blastocysts*
B Zygote
C Morula
D Gastrula
E Neurula
Microspecimen analysis of child's finger skin revealed that epidermis has signs of
inadequate development. What embryonal leaf was damaged in the process of
development?
A Ectoderm*
B Mesoderm
C Entoderm
D Mezenchym
E Ectomezenchym
During gastrulation the Hensen's node remained underdeveloped in the embryo.
Which axial organ will slow down its development?
A Chord*
5
B Neural crests
C Neural groove
D Neural tube
E Mantle layer of the neural tube
On a section of a germ of the person taken from spontaneous abortion, we see
embryonic disk in which two layers are distinguished, - еndo-and еctoderm. At what
stage of embryonic development there was an embryo?
A.Blastula formation
B.Histogenesis
C.Progenesis
D.Gastrulation*
E.Organogenesis
One of the critical periods of human embryogenesis is implantation of a germ in a
wall of a uterus during 5th-7th days. What mechanism of gastrulation occurs in embryo
during this period?
A. Еpibolia
B. Migration
C. Delamination*
D. Invagination
E. Neurulation
During gastrulation in a germ it was insufficiently generated primary node. What
structure of developed embryo’s body will be broken?
A. Chord *
B. Nervous crests
C. Ectoderm
D. Nervous tube
E. Mantile layer of a nervous tube
Early gastrulation of a human germ occurs by delamination of embryoblast. From what
structure will be formed nervous system?
A. From hypoblast
B. From trophoblast
C. From epiblast*
D. From chord
E. From primitive segments
Implantation of human blastocyst begins. How the period of embryogenesis refers to,
what begins simultaneously with implantation?
A. Histogenesis
B. Invagination
C. Differentiation
D. Gastrulation *
E. Segmentation
In newborn infringement of development of a myocardium is diagnosed. To what
infringement of development of embryonic source this pathology is connected?
A.Ectoderm
B.Entoderm
C.Splanchnopleure*
D. Mesenchyma
E.Somatopleure
At Addison illness’ patient hyperpigmentation of integuments is observed. It connects
with a generality of sources of melanocytes’ development and suprarenal gland’s
medulla. What is a source of their development?
A.Ectoderm
6
B.Mesoderm
C.Mesenchyma
D. Nervous crest*
E.Endoderm
At the newborn boy (10th days) numerous defects of development of skeleton are
observed. What infringement of embryonic development a rudiment has most
authentically led to occurrence of the given pathology?
A. Miotomes
B. Mesenchyma
C. Dermatome
D. Еctoderm
E. Sklerotome*
In experiment on frog’s germ the external germinal layer - еctoderm is destroyed.
What morphological structure from listed further will not develop at the given germ?
A. Bone tissue
B. Blood
C. Epidermis *
D. Cartilage tissue
E. Muscle tissue
In experiment at germ of rabbit is destroyed myotome. Infringement of development
of what structure will be observed at the given germ?
A. Epidermis
B. Bone tissue
C. Connective tissue
D. Smooth muscles
E. Skeletal muscles *
On histological section it is visible a hen’s germ at a stage of differentiation of
mesoderm to somites, segment legs and splanchnotome. From what material does the
skeleton develop?
A. Myotome
B. Dermatome
C. Celom
D. Splanchnotome
E. Sklerotome*
In course of a conditional experiment the development of mesenchyma cells was
completely inhibited. Development of the following muscular tissue will be disturbed:
A Smooth muscular tissue *
B Neural muscular tissue
C Epidermal muscular tissue
D Cardiac muscular tissue
E Skeletal muscular tissue
An embryo displays disturbed process of dorsal mesoderm segmentation and somite
formation. What part of skin will have developmental abnormalities?
A Derm *
B Hair
C Sebaceous glands
D Epidermis
E Sudoriferous glands
Study of the biopsy material of an embryo revealed a zone of developmental
abnormality in a somite. The zone was located close to the endoderm and the
notochord. What formations may have abnormal development in case of pregnancy
continuation?
7
A Skeletal tissues *
B Genito-urinary system
C Skeletal striated muscle tissue
D Cardiac striated muscle tissue
E Fibrous connective tissue of skin
8.
Provisional
organs. Placenta.
Umbilical cord.
By producing a number of hormones placenta plays a part of temporary endocrine
gland. What hormone may be detected in woman's blood on the third or the forth day
after begin of implantation, that is used in medicine for early pregnancy detection?
A Chorionic gonadotropin*
B Somatostatin
C Progesterone
D Vasopressin
E Oxytocin
At microscopic research of environments of a germ it is defined determined chorion.
What basic function is provided with this structure?
A. Metabolism between mother’s blood and fetus’ blood*
B. Haemopoietic function
C. Production of amniotic water
D. Formation of primary reproductive cells
E. Differentiation of the lymphocytes
Ultrasound examination of pregnant women was diagnosed with polyhydramnios. In
violation of which structure can be attributed this pathological condition?
A Amnion*
B Chorion
C Placenta
D Yolk sac
E Allantois
The examination of amniotic fluid obtained by amniocentesis (amnion puncture) there
were found cells which nucleuses contain sex chromatin (Barr’s body). What could
this mean?
A The development of male fetus
B Development of female fetus*
C Genetic disturbances in fetal development
D Trisomy
E Polyploidy
9.
10.
Control of
mastering of the
submodule 1.
Basic principles
of organization of
tissues. Epithelial
tissues
Histological examination of a tissue sample revealed that the tissue have no blood
vessels, and the cells were packed tightly together making layers. Specify this tissue:
A Epithelial*
B Cartilaginous
C Osseous
D Nervous
E Muscular
In the patient it is found out hyperemia of skin. Borders of the damaged injured sites
of skin are precisely outlined, have the wrong form and painless. What type of
epithelia is struck with a mycosis?
A. Simple
B. Stratified cuboidal
C. Keratinazed stratified*
D. Transitional
E. Pseudostratified columnar
8
To the man 48 years old with the diagnostic purpose it is lead is carried out biopsy of a
stomach tumor located in the mucosa. Immunomorfological research of bioptate has
shown presence in cytoplasm of cells keratin filaments. The most probable source of
development of a tumor is:
A. Epithelial tissue*
B. Areolar connective tissue
C. Dense connective
D. Muscle tissue
E. Nervouse tissue
At the patient with a dry pleuritis noise of friction of pleura is listened. At what defeat
of type of epithelia this symptom is marked?
A. Stratified epithelium
B. Simple squamous*
C. Transitional
D. Cuboidal
E. Cilindrical
In the experiment the structure of contact between epithelial cells affected. What is
the function of epithelium suffered?
A Mechanical *
B Absorbing
C Vitamin "D"-producing
D Secretory
E Excretory
Tumor of the pericardium was diagnosed in the man. What epithelium is a source of
tumor development?
A Simple squamous*
B Pseudostratified epithelia
C Transitional
D Stratified epithelium keratinized
E Stratified epithelium non-keratinized
The secretory part of apocrine sweat glands contains myoepithelial cells. What is the
function of these cells?
A Contractility*
B Secretory.
C Security.
D Regeneration.
E Maintance.
A scheme presents an exocrinous gland that has unbranched excretory duct with a
terminal part in shape of a saccule openining into the duct. How this gland is called
according to the morphological classification of exocrine glands?
A Simple unbranched alveolar *
B Compound branched alveolar
C Simple branched tubular
D Compound unbranched alveolar
E Compound unbranched alveolar tubular
11
Blood and lymph.
A 30 y.o. man was irradiated with approximately 3 Gy. What blood changes will be
revealed 8 hours after exposure to radiation?
A Lymphopenia*
B Leukopenia
C Granulocytopenia
D Thrombocytopenia
E Anemia
9
In course of an operation on account of a granuloma in the area of the right upper
incisor a patient began to bleed. The hemorrhage was stopped just only 3 hours later.
The patient's anamnesis contains information about chronic lymphatic leukemia.
What is the most probable cause of hemorrhage?
A Thrombocytopenia*
B Thrombocytopathia
C Lymphocytosis
D Leukopenia
E Eosinophilia
As a result of a road accident a 26-year-old man is in the torpid phase of shock. Blood
count: leukocytes - 3,2x109/l. What is the leading mechanism of leukopenia
development?
A Leukocyte redistribution in the bloodstream*
B Leukopoiesis inhibition
C Faulty release of mature leukocytes from the bone marrow into the blood
D Leukocyte destruction in the hematopietic organs
E Increased excretion of the leukocytes from the organism
Examination of mountain climbers who have spent a long time in a high-altitude
region revealed increase of erythrocyte number (over 6x1012/l) and haemoglobin
concentration (over 170 g/l). What mechanism caused this phenomenon?
A Intensified production of erythropoietin by the kidneys*
B Weakening of erythrocyte haemolysis in bloodstream
C Improved ability of tissue for oxygen utilization
D Intensified processes of anoxic energy production
E Weakening of intracellular erythrocyte haemolysis
In smear of peripheral blood among leukocytes prevalate spherical cells with the
segmented nucleus. Fine granularity in their cytoplasm colorated in blue and red
colours. How these cells refer to?
A. Young neutrophills
B. Basophills
C. Acidophills
D. Segment-shape-nucleus neutrophills *
E. Моnocytes
At the analysis of blood the reduced maintenance hemoglobin’s contents is revealed.
What function of blood will be disturbed thus?
A. Transport of hormones
B. Transport of gases *
C. Maintenance of immunity
D. Blood regeneration
E. Transport of nutrients
Inflammation is characterized by reaction of blood capillaries on a site of damage,
reduction of blood circulation, increase of permeability of the vessels’ wall. To what
cells, mentioned below, possesses a leaguing role in it?
A. To macrophages
B. To fibroblasts
C. To plasma cells
D. To acidophills
E. To basophills*
In the analysis laboratory assistant has the additional conclusion that the blood belongs
to the women. Features of the structure of what formed elements allow making a
conclusion?
A Neutrophilic leukocytes*
B Erythrocytes
10
C Lymphocytes
D Monocytes
E Basophilic leukocytes
In smear of peripheral blood among leukocytes prevalate spherical cells with the
segmented nucleus. Fine granularity in their cytoplasm colorated in blue and red
colours. How these cells refer to?
A. Young neutrophills
B. Basophills
C. Acidophills
D. Segment-shape-nucleus neutrophills *
E. Моnocytes
At the analysis of blood the reduced maintenance hemoglobin’s contents is revealed.
What function of blood will be disturbed thus?
A. Transport of hormones
B. Transport of gases *
C. Maintenance of immunity
D. Blood regeneration
E. Transport of nutrients
Inflammation is characterized by reaction of blood capillaries on a site of damage,
reduction of blood circulation, increase of permeability of the vessels’ wall. To
what cells, mentioned below, possesses a leaguing role in it?
A. To macrophages
B. To fibroblasts
C. To plasma cells
D. To acidophills
E. To basophills*
In the analysis laboratory assistant has the additional conclusion that the blood belongs
to the women. Features of the structure of what formed elements allow making a
conclusion?
A Neutrophilic leukocytes*
B Erythrocytes
C Lymphocytes
D Monocytes
E Basophilic leukocytes
At histochemical research of leukocyte in blood smear there were found cells in which
contain granules with histamine and heparin. What are cells?
A Basophils*.
B Neutrophils.
C Eosinophils.
D Monocytes.
E Erythrocytes
At transplantation of organ there was detected transplant rejection. Which blood cells
provide this process?
A T – killer *
B T-helper
C T-suppressor
D T-lymphocytes-O
E T-lymphocytes-memory
Large cells with basophilic cytoplasm and bean-shaped nucleus is seen in a blood
smear. The cell is the largest among the visible cell in the field of view. What is a
cell?
A Monocytes*
11
B Macrophage
C Plasmocyte
D Basophils
E Small lymphocyte
In the experiment B lymphocytes were market in blood. Foreign protein was entered
the animal under the skin. What cells in the connective tissue will contain this mark?
A Plasma cells *
B T lymphocytes
C Macrophages
D Basophils
E Fibroblasts
Numerous plasma cells were found in the blood girl 16 years old, suffering from an
autoimmune inflammation of the thyroid gland. With the proliferation and
differentiation of what blood cells cause increase of plasmocyte?
A B lymphocytes*
B T-helper
C Mast cells
D T-killer
E T-suppressor
A student is given two smears preparations. One - the whole field of view covered by
erythrocytes, the second - defined blood cells in varying degrees of maturity. What
kind of smear?
A Blood and red bone marrow *
B Blood and lymph
C Blood frogs and human blood
D Blood smear and yellow bone marrow
E Smear of yellow and red bone marrow
At research of the patient's blood smear there were found the cells wich make up the
0.5% of total leukocytes and have S-shaped nucleus and blue granules in the
cytoplasm. What are these cells?
A Basophils*
B Neutrophils
C Eosinophils
D Monocytes
E Lymphocytes
Live vaccine is injected into the human body. Increasing activity of what cells of
connective tissue can be expected?
A Plasma cells and lymphocytes*
B Macrophages and fibroblasts
C Pigmentocytes and pericytes
D Adipocytes and adventitious cells
E Fibroblasts and labrocytes
In the blood of a 26-year-old man it was revealed 18% of erythrocytes of the
spherical, ball-shaped, flat and thorn-like shape. Other erythrocytes were in the
form of the concavo-concave disks. How is such phenomenon called?
A Physiological anisocytosis
B Pathological poikilocytosis
C Physiological poikilocytosis *
D Pathological anisocytosis
E Erytrocytosis
12
Myeloid and
lymphoid tissues
In myeloid tissue punctate child 6 years cells are found. In these cells there were
remove the nucleus during the process of differentiation. What kind of
12
of haemopoiesis.
Myelopoiesis and
lymphopoiesis.
hematopoiesis, characterized by morphological changes in the data.
A Erythrocytopoiesis *
B Trombotcytopoiesis.
C Granulcytopoiesis
D Lymphocytopoiesis
E Monocytopoiesis
At histological examination of biopsies of red bone marrow there were found the
cells of granulocytic group. What changes take place in the nucleus during
differentiation of these cells?
A Segmentation*
B Poliploidiya
C Piknoz
D Enucleation
E Increasing the size
At electron micrographs of red bone marrow ther were the megakaryocytes wich have
demarcation channels in the peripheral part of the cytoplasm. What is the role of
these structures?
A Formation of platelets *
B Increasing the surface area of the cells.
C Increase the number of ion channels.
D Division cell
E Destruction of cells
In punctate of red bone marrow ther were found a significant decrease in the number
of megakaryocytes. What changes in peripheral blood is accompanied?
A Decrease the number of platelets*
B Increasing the number of lymphocytes
C Increase the number of platelets
D Decrease the number of lymphocytes
E Decrease the number of monocytes
During postembryonal haemopoiesis in the red bone marrow the cells of one of the
cellular differons demonstrate a gradual decrease in cytoplasmic basophilia as well as
an increase in oxyphilia, the nucleus is being forced out. Such morphological
changes are typical for the following haemopoiesis type:
A Erythropoiesis *
B Lymphopoiesis
C Neutrophil cytopoiesis
D Eosinophil cytopoiesis
E Basophil cytopoiesis
13
Fibrous
connective
tissues.
A specimen of connective tissue of derma was stained with Sudan III and
hematoxylin. There are clusters of big polygonal cells that turned orange. Their nuclei
are flattened and located on periphery. What tissue is it?
A White adipose*
B Brown adipose
C Reticular connective
D Hyaline cartilaginous
E Lamellar osseous
To the cosmetician the patient with the request has addressed to deprive his tattoo on a
shoulder. What substance which contains in a connective tissue, limits distribution of
dye and makes possible such kind of "painting"?
A. Geparin
B. Fibronektin
C. Gamma-globulin
D. Elastin
13
E. Hyaluronic acid*
At healing a wound in a site of defect of tissue develops scar from connective tissue.
What cells provide this process?
A. Fibrocytes
B. Маcrophages
C. Fibroblasts*
D. Lymphocytes
E. Меlanocytes
On the histologic section of areolar connective tissue rather big cells filled basophilic
granularity are found; with histochemical methods it is established, that granules
contain heparine and histamine. What’s type of cells?
A. Аdipocytes
B. Fibroblasts
C. Маcrophages
D. Plasma cells
E. Mast cells*
As a result of contact on manufacture with Cr-connections at the woman has arisen
allergic dermatitis of both hands. What cells of the skin have mainly taken part in
realization of this disease?
A. Mast cells*
B. Plasma cells
C. Маcrophages
D. Neutrophills
E. Lymphocytes
The esophageal constriction occurred after a chemical burn due to its local scar
formation. What cells of loose connective tissue involved in the formation of scars?
A Mature specialized fibroblasts*
B Young fibroblasts
C Fibrocyte
D Myofibroblasts
E Fibroclast
As a result of thrombosis of the left coronary artery the groups of contractile
cardiomyocytes were damaged. What type of cells will take part in reparative
regeneration in the area of damage?
A Fibroblasts*
B Cardiomyocytes
C Miosymplast
D Miosateliocyte
E Smooth myocytes
In the experiment B lymphocytes were labeled. The animal was injected subcutaneous
a foreign protein. What cells of the connective tissue will contain this label?
A Plasmocyte *
B T lymphocytes
C Macrophages
D Basophils
E Fibroblasts
During the training an athlete was injured lower extremity. Traumatologist diagnoses
the rupture of tendon. What type of connective tissue does tendon belongs?
A Dense regular connective tissue*
B Dense irregular connective tissue
C Loose connective tissue
D Reticular tissue
14
E Cartilage
With age, the human skin is changed, its elasticity decreases. What elements of
connective tissue provide its elasticity?
A Collagen and elastic fibers*
B Ground substance
C cells of the epidermis
D cells of connective tissue
E Reticular fibers
A lot of stem cells of red bone marrow were destroyed in the experiment.
Development of what populations of cells in the loose connective tissue is inhibited?
A Macrophages*
B Fibroblasts
C Pigment cells
D Lipocyte
E Pericytes
Decreased blood supply to the organs causes hypoxia that activates fibroblasts
function. Volume of what elements is increased in this case?
A Intercellular substance *
B Vessels of microcircular stream
C Nerve elements
D Parenchymatous elements of the organ
E Lymphatic vessels
14
Development and
structure of
cartilage tissue
During the experimental analysis of chondrohistogenesis a sclerotome was damaged.
What cells will it make impossible to differentiate?
A Chondroblasts*
B Smooth myocytes
C Myoblasts
D Fibroblasts
E Epidermocytes
As a result of a chest trauma the costal cartilage was damaged. The cartilage
regenerated due to the following layer of perichondrium:
A Chondrogenic*
B Fibrous
C Elastic
D Collagen
E Sharpey's fibers
At electronomicroscopic research of the hyaline cartilage’s cells with well advanced
rough endoplasmic reticulum, Golgy complex come to light. What function is
carried out with these cells?
A. Secretion of the lipids
B. Desruction of intercellular substance of a cartilage
C. Keeping of the glycogen
D. Formation of intercellular substance*
E. Nutrition of the cartilage tissue
On histological preparation of cartilage are isogenic group of cells. What are the
initial cells in the formation of these groups?
A Chondrocytes I type *
B Chondroblasts.
C Prechondroblasts.
D Chondrocytes type II.
E Chondrocytes type III
15
Articular cartilages don’t have perichondrium. What type of these cartilages’ growth
are in the process of regeneration?
A Interstitial*
B Appositional
C By imposing
D Appositional and interstitial
E Does not react
15
Structure of bone
tissues.
The symptoms of regeneration process (callus) on the place of fracture were
revealed in the histologic specimen of tubular bone. What tissue forms this structure?
A Fibrous bone tissue*
B Loose connective tissue
C Reticular tissue
D Epithelial tissue
E Lamellar bone tissue
A histological specimen presents the tissue that contains cells having no processes and
a few tens of nuclei each. One of cell surfaces has a corrugated zone that provides
secretion of hydrolytic elements. What tissue is it?
A Bone tissue*
B Cartilage tissue
C Epithelial tissue
D Nerve tissue
E Muscle tissue
Calcification of the intercellular substance of bone tissue is accompanied by the
deposition of hydroxyapatite crystals along the collagen fibers. This process requires
the presence of alkaline phosphatase in the intercellular substance. What cell produces
this enzyme?
A Osteoblast*
B Osteocyte
C Osteoclast
D Chondroblast
E Chondrocyte
The resorbtion (destruction) of bones is found in the patient. To what hyperactivity
of bone cells it is connected?
A. Оsteocytes
B. Оsteoblasts and osteoclasts
C. Оsteocytes and osteoclasts
D. Оsteoblasts
E. Оsteoclasts*
In section of the tissue which contains cells without processes is submitted and
everyone has ten nucleus. One of surfaces of cells has a goffered zone through
which there is a secretion of hydrolyzed enzymes. What tissue is submitted in
section?
A. Cartilage tissue
B. Epithelial tissue
C. Nervouse tissue
D. Bone tissue*
E. Muscle tissue
There is an increased loss of bone mass in older people, which shows the
development of osteoporosis. Activation of what type of bone cells leads to the
development of this disease?
A Osteoclasts*
B Osteoblasts
C Macrophages
16
D Basophils
E Osteocytes
16
17
Development of
bone tissues.
Development and
structure of
muscles tissues
On the Ro-gram of skeleton it is visible, that all three parts of pelvic bones are
separated by intervals which answer an invisible cartilage on Ro-grams. It is typical of
what age?
A. To 25 years old
B. To 30 years old
C. To 16 years old*
D. Till 40 years old
E. To 50 years old
In course of indirect osteogenesis of tubular bone tissue a plate is formed between
epiphyseal and diaphyseal ossification centres that provides further lengthwise growth
of bones. What structure is it?
A Metaphyseal plate *
B Osseous cuff
C Osseous plate
D Osteon
E Layer of interior general plates
The regeneration process of damaged skeletal muscles іs very slow. What elements of
muscle fiber take part in the process of regeneration?
A Myosatellitocytes*
B Myoblasts
C Smooth myocytes
D Myofibroblasts
E Myoepithelial cells
In clinical conditions trauma of skeletal muscles is diagnosed for the patient. The
fibers recycles slowly for the account:
A. Mitosis of the myosatelitocytes*
B.Division of the muscle fibres
C.Differentiation of fibroblasts
D. Increases in quantity amount of myofibrilles
E.Increases in quantity amount of sarcoplasm
In a phase of contraction of a myocardium (systole) in sarcoplasm of cardiomyocytes
concentration of calcium’s ions sharply increases. What structures take part in
deposition of calcium’s ions?
A. T - system
B. lysosomes
C. Ribosomes
D. L-system *
E. Nucleoles
At research of a cross-section of skeletal muscle fibers after mechanical trauma
destruction thick myofilaments is observed. Where pathological changes will be
located?
A. In a disk І
B. In a disk A*
C. In half of disk A
D. In disk A and in disk I
E. In half of disk I
At micropreparations of submandibular salivary glands there were the cells around
the terminal parts and ducts that cover the serocyte and called mioepiteliocyte.
What kind of tissue these cells belong to?
A Muscle*
B Epithelial
17
C Nervous
D Connective with special properties
E Loose connective
In histological preparation tissue is presented, the basic structural unit of which is fiber
that consists of symplast and satellite cells, covered by a basement membrane. For
what tissue this structure is characterized by?
A Skeletal muscle tissue *
B Smooth muscle tissue.
C Cardiac muscle tissue.
D Loose connective tissue.
E Reticular tissue.
At research of striated muscle fibers after the influence of hydrolytic enzymes
fracture of thin myofilaments was observed. Which structures have been damaged?
A Actin myofilaments *
B Tonofibrils
C T - Systems
D Sarcoplasmic reticulum
E Myosin miofilaments
Patient with injured muscles of the lower extremities was admitted to the
traumatological department. Due to what cells is reparative regeneration of the
muscle fibers and restoration of the muscle function possible?
A Satellite-cells*
B Myoblasts
C Myofibroblasts
D Fibroblasts
E Myoepithelial cells
The development of mesenchymal cells was inhibited in experiment. Development
which muscle tissue will be disturbed?
A Smooth muscle tissue*
B Muscle tissue neural origin
C Muscle tissue epidermal origin
D Cardiac muscle tissue
E Skeletal muscle tissue
Negative environmental factors have caused the dysfunction of myosatellite cells.
What function of the whole muscle fibre is likely to be changed in this case?
A Regeneration *
B Contraction
C Trophism
D Contractile thermogenesis
E Relaxation
18
Nervous tissue.
Neurons.
Neuroglia.
A sensitive neural ganglion consists of roundish neurocytes with one extension that
divides into axon and dendrite at some distance from the perikaryon. What are
these cells called?
A Pseudounipolar *
B Unipolar
C Bipolar
D Multipolar
E Apolar
19
Nervous fibres.
Regeneration of
nervous fibres.
Nervous endings.
At traumatic damage probably development of degeneration of nervous fibers which is
accompanied by breakage of axial cylinders, disintegration of myelin. Due to what
nervous structures there is a renewal of myelin at regeneration?
A. Gliocytes
18
Classification and
structure.
B. Shwann cells*
C. Peryneurium
D. Еndoneurium
E. Аstrocytes
In conditional experiment action of toxic substance breaks the mechanism of the
nervous impulse conduction. What structure provides performance of the given
function?
A. Neurolemma
B. Synaps *
C. Neurofibrills
D. Мitochondria
E. Nissle substance
20
21
Control of
mastering of the
submodule 2.
Final test control
of the module 1.
MODULE 2
№
TOPIC
1
Nervous system.
Spinal cord.
Spinal ganglion.
TEST
After amputation of the left top finiteness of the patient long time felt a strong pain
in it. What mechanism of formation of painful unhealthy sensations is most probable
in this case?
A. Fantomic *
B. Reflectory
C. Kausalgic
D. Hypoproduction of encephalines
E. Hyperproduction of encephalines
In a histological preparation the organ which will consist of grey and white substance
is determined. The grey substance settles down in the center and will consist from
radicular, associative neurons. Name body for which the given morphological
attributes are characteristic.
A. Big hemispheres
B. Brain
C. Cerebellum
D. Spinal ganglia
E. Spinal cord*
In a histological preparation parenchyma of organ it is submitted by nervous fibers in
which are determined pseudo-unipolar neurons. Neurons’ bodies are covered by glial
and connective shalls, placed by groups. Named organ that has given morphological
attributes.
A. Epiphysis
B. Spinal cord
C. Spinal ganglia *
D. Brain
E. Cerebellum
At a mechanical trauma the site of a peripheral nerve in length more than 10 sm is
damaged injured. In result impellent activity of the top finiteness is broken.
Allotransplantation of criopreserved nerve is offered to the patient. What glial cells
will take part in regeneration and maintenance of trophic the damaged injured site?
A. Microglial cells
B. Fibrous astrocytes
C. Neurolemmocytes *
D. Ependimogliocytes
19
E. Leucocytes
As a result of a trauma a patient has damaged anterior roots of spinal cord. What
structures have been affected?
A Axons of motoneurons and axons of neurons of lateral horns *
B Central processes of sensitive neurons of spinal ganglions
C Peripheral processes of sensitive spinal ganglions
D Axons of neurons of lateral horns
E Dendrites of neurons of spinal ganglions
2
Nervous system.
Cerebrum.
Cerebellum.
In a specimen that was coloured by method of silver impregnation some piriform cells
with 2-3 evident dendrites were found. What structure is being analysed?
A Cerebellar cortex*
B Spiral organ of middle ear
C Retina
D Cerebral cortex
E Spinal ganglion
One of sections of central nervous system has layerwise arrangement of neurocytes.
Among them there are cells of the following forms: stellate, fusiform, horizontal,
pyramidal. What section of central nervous system is this structure typical for?
A Cortex of cerebrum*
B Spinal cord
C Cerebellum
D Medulla oblongata
E Hypothalamus
On preparation organ of nervous system which contains grey and white substance is
submitted. The grey substance is placed on periphery. Neurons form three layers:
molecular, ganglionar and granular. What organ is on preparation?
A. Brain cortex
B. Spinal cord
C. Spinal ganglia
D. Pons
E. Cerebellum cortex*
In a histological preparation the cut of precentral girus of the hemispheres is
submitted. Specify, what layers are most advanced in this zone?
A. Molecular, pyramidal, ganglionar
B. Molecular
C. External and internal granular
D. Molecular and the layer of polymorphic cells
E. Pyramidal, ganglionar and a layer of polymorphic cells *
The alcoholic intoxication, as a rule, is accompanied by infringement of coordination
of movement and balance, as a result of damage of structural elements of a
cerebellum. Function of what cerebellum’s cells is broken first of all?
A. Stellate cells
B. Pyramidal cells
C. Fusiform cells
D. Pear-shaped cells*
E. Granular cells
In histological section, impregnated by salts of silver, determined the cerebellum
cortex which contains pear-shaped, bascet-shape, stellate neurons, cells-grains. Named
neurocytes which are part of the ganglionar layer.
A. Pear-shaped*
B. Stellate, pyramidal
C. Cells - grains, large stellate
20
D. Large stellate and bascet-shape
E. Bascet-shape, small and large stellate
In a histological section the body which will consist of grey and white substance is
determined. The grey substance settles down on periphery and has 6 layers:
molecular, external granular, pyramidal, internal granular, ganglionar and the layer of
polymorphic cells. Determine structure to which possesses the given morphological
attributes.
A. Cerebellum
B. Brain*
C. Bone marrow
D. Spinal ganglia
E. Spinal cord
One of the parts of central nervous system has stratified arrangement of neurocytes.
Among them there are cells of the following shapes: stellate, fusiform, horizontal,
pyramidal. What section of central nervous system is this structure typical for?
A Cortex of cerebrum *
B Spinal cord
C Cerebellum
D Medulla oblongata
E Hypothalamus
3
Vegetative
nervous system.
Long stay in conditions of high temperature caused thirst in the person. The signal
system from what receptors, first of all, has caused its development?
A. Sodium receptors of hypothalamus
B. Osmoretseptors of hypothalamus*
C. Baroretseptors of aorta’s arch
D. Glycoreceptors of hypothalamus
E. Osmoretseptors of liver
4
Sense organs.
Visual organ.
As a result of punctate retinal hemorrhage a patient lost ability to see objects in the
centre of visual field. In what part of retina did the hemorrhage take place?
A Yellow spot*
B Ciliary part of retina
C Iris
D Blind spot
E Vascular membrane
In histological preparation of the wall of an eyeball the structure which will consist of
three neurons is determined. Bodies of these neurons form external, internal
nuclear and ganglionar layers. What formation education of an eye has such
morphological structure?
A. Retina *
B. Iris
C. Sсlera
D. Vascular shall
E. Ciliar body
To the patient transplantation of the cornea is executed. What features of cornea’s
structure allow to hope for her it reparation, instead of tearing away?
A. Innervation
B. Multilayered epithelia
C. Absence of blood and typical lymphatic vessels*
D. Presence of the connective tissue
E. Simple epithelia
In an electronic microphoto the cell of neural origins is submitted. The terminal part
of cells’ dendrite has cylindrical shape and will consist from 1000 closed
21
membranouse disks. What it for a cell?
A. Rod-shape cell *
B. Neuron of spinal cord
C. Neuron of spinal ganglia
D. Neuron of the hemispheres
E. Cones-shape cell
At the patient at inspection infringement of perception recognition of green color is
revealed. What absence of cells of retina causes the given infringement of sight?
A. Cones-shape cells*
B. Rod-shape cells
C. Pigmental epithelia
D. Bipolar neurons
E. Ganglionar neurons
During examining ophthalmologist found out that the patient does not distinguish
blue and green, with normal perception of other colors. Dysfunction of what retinal
structures is that?
A Cones-shape cells *
B Rod-shape cell
C Bipolar neurons
D Amacrine neurons
E Horizontal neurons
Patient’s ciliary body was damaged. What function of eye was affected?
A Accommodation*
B Light-conductive
C Light-refracting
D Protection
E Trophic
The increased intraocular tension is observed in the patient with glaucoma.
Secretion of aqueous humor by the ciliar body is normal. Injury of what structure of
the eyeball wall caused the disorder of flow-out from the anterior chamber?
A Venous sinus*
B Ciliar body
C Choroid
D Ciliary muscle
E Back epithelium of cornea
At insufficiency of vitamin A at the person twilight sight is broken. Specify cells to
which poses given receptor function.
A. Rod-shape cells *
B. Cones-shape cells
C. Bipolar neurons
D. Horizontal neurons
E. Ganglionar nervous cells
An infectious disease caused contractive activity of muscles that contract and dilate
eye pupil (paralytic state). What functional eye system was damaged?
A Accomodative *
B Dioptric
C Ancillary
D Photosensory
E Lacrimal apparatus
A histological specimen of the eyeball shows a biconvex structure connected to the
ciliary body by the fibers of the Zinn's zonule and covered with a transparent
capsule. Name this structure:
22
A Lens *
B Vitreous body
C Ciliary body
D Cornea
E Sclera
5
Sense organs.
Auditory and
vestibular organs.
The animal well keeps an equilibrium antigravitational pose at inclinations of the head,
at movements with linear acceleration, however frequently loses balance at
movements with angular acceleration. What infringement of functions of structures
is the reason of it?
A. Crista*
B. Macula
C. Nervus vestibulocohlearis
D. Lateral vestibular nucleus
E. Vestibulospinal way
In a histologic preparation the receptor zone of sensoepithelial sense organ is defined.
Cells of the given zone are on basal membrane and include the following kinds:
external and internal sensitive, external and internal phalangeal, cells - columns,
external boundary and external supporting. What sense organ posesses given
receptor zone?
A. Smell organ
B. Taste organ
C. Vestibular organ
D. Auditory organ*
E. Visible organ
At the patient who accepted the big dozes of streptomycin, there has come stepped
loss of hearing. What function of cells of an internal ear has been damaged injured in
this case?
A. Deiter’s cells
B. Phalanx-cells
C. Cells - columns
D. Sensoepithelial cells *
E. Fibroblasts
A histological specimen presents a receptor zone of a sensoepithelial sense organ.
Cells of this zone are placed upon the basal membrane and include the following types:
external and internal receptor cells, external and internal phalangeal cell, stem
cells, external limiting cells and external supporting cell. The described receptor
zone belongs to the following sense organ:
A Visual organ
B Auditory organ *
C Gustatory organ
D Equilibrium organ
E Olfactory organ
Ampullary crests were damaged as a result of head trauma in men 32 years old.
Perception of what stimuli was disrupted?
A Angular acceleration*
B Vibration
C Gravitation
D Linear acceleration
E Vibration and gravitation
6
Sense organs.
Olfactory and
taste.
In the patient with an acute rhinitis it is found out hyperemia and formation of mucus
in nasal cavity is raised increased. What activity of epithelial cells of mucosa is raised
increased?
A. Endocrine
23
B. Goblet cells *
C. Basal
D. Cells with villies
E. Cilindrical
In an electron microphoto the cell of neural origins which is in structure epithelia of
mucosa is submitted. Distal part of a peripheral shoot of the cell has knob-shape
from which depart 10-12 cilia. What it for a cell?
A. Olfactory cell *
B. Bipolar neuron of spinal ganglia
C. Sensitive taste cell
D. Rod-shape cell
E. Cones-shape cell
The boxer does not sense smell after nose trauma. Damage of what cell can lead to
loss of smell?
A Neurosensory cells*
B Supporting epithelial cells
C Basal epithelial cells
D Ciliated epithelial cells
E Goblet cells
After prolonged inflammation of the nasal mucosa there were observed changes in the
epithelium. What type of epithelium has changed?
A Pseudostratified columnar*
B Simple squamous
C Stratified squamous
D Stratified cuboidal
E Stratified columnar
As a result of injury of male’s nose there was damaged mucosa covering the upper
part of the superior nasal conchal. What is the result of this?
A Violation of the perception of smell*
B Violation air humidification
C Violation of secretory activity of goblet cells
D Violation warming air
E Violation of warming and humidification
7
Skin. Skin glands. End-pieces of apocrinic sweat glands contain myoepithelial cells. What function of
Hair and nails
these cells?
A. Contraction *
B. Secretory
C. Protective
D. Regeneration
E. Supporting
In a histological preparation the organ which wall has the layered structure, covered
keratinized epithelium is submitted. Under basal lamina there is loose connective
tissue which formated papilles. Dense connective tissue forms deep layer. What organ
has the given morphological attributes?
A. Skin *
B. Tongue
C. Esophagus
D. Tonsils
E. Uterus
With age skin of human changed, which can be shown by reduction of its elasticity.
What elements of a connective tissue most of all provide its elasticity?
A. Ground substance
24
B. Elastic fibers *
C. Epidermal cells
D. Collagen fibers
E. Reticular fibers
In biopsy material of skin in epidermis polyhedral cells which have granules of dark
brown color in cytoplasm are found out. What are these cells?
A. Lymphocytes
B. Macrophages
C. Keratinocytes
D. Merkel’s cells
E. Melanocytes*
The patient with hepatites complains of an itch of skin. To what function of
appendages of skin this symptom is connected?
A. Excretory of sweat glands*
B. Protective of sebaceous glands
C.Termoregulatory of sweat glands
D. Protective of hair
E. Protective of nails
Between epidermal cells there are many types of cell with different shape and
function. Which of them are derivatives of red bone marrow?
A. Langergans’ cells*
B. Merkel’s cells
C. Melanocytes
D. Keratinocytes
E. Pennet’s cells
The skin with damage of deep layer is injured. Due to what cells activity
regeneration of the given layer will take place?
A. Erithroblast
B. Macrophages
C. Lymphocytes
D. Neuroblasts
E. Fibroblasts*
At patient А., 12 years, on skin white stain which have no pigment. Stain has
appeared after 10 years, constantly increase in sizes. What absence of skin’s cells has
led to occurrence of this problem?
A. Adipocytes
B. Melanocytes*
C. Fibroblasts
D. Plasma cells
E. Mast cells
In a limited area of the epidermis due to injury the layers are absent to germinativum.
Name the cells that serve as the main source of its regeneration.
A Basal layer cells *
B Cells of strarum spinosum.
C Cells of strarum granulosum.
D Cells of strarum corneum.
E Cells of strarum lucidum.
A patient complains of dryness of head skin, itching, fragility and loss of hair. After
examination he was diagnosed with seborrhea. Disturbed activity of which cells
caused this condition?
A Cells of sebaceous glands *
B Cells of sudoriferous glands
25
C Epithelial cells
D Adipocytes
E Melanocytes
Study of fingerprints is used by criminalists for personal identification as well as for
diagnostics of genetic abnormalities, particularly Dawn's disease. What layer of skin
determines individuality of fingerprints?
A Dermal papillae *
B Stratum corneum epidermidis
C Reticular
D Stratum lucidum epidermidis
E Basal
8
Circulatory
system. Heart
After physical loading arterial pressure of blood has risen increased. Why?
A. Increasing quantity amount of functioning capillaries
B. Increasing of maintenance contents of Kalium ions in blood plasma
C. Increasing of minute volume of blood circulation*
D. Increasing of maintenance contents of water in plasma
E. Increasing of quantity amount of hemoglobin
As a result of the thrombosis of an artery the destruction of contractive
cardiomyocytes has taken place. Due to what cells there will be reparative
regeneration in a zone of damage?
A. Fibroblasts *
B. Cardiomyocytes
C. Myosymplast
D. Myosatelitocytes
E. Smooth muscle cells
The serous liquid collects in pericardial cavity of the patient with pericarditis. With
infringement of what cell’s activity of the pericardium this process is connected?
A. Mesothelial cells*
B. Endothelial cells
C. Smooth myocytes
D. Fibroblasts
E. Macrophages
At micropreparations of heart ther were distinguished rectangular shape cells with a
centrally located nucleus, developed myofibrils, linked intercalated discs. Since
these cells is associated function:
A Contraction of the heart*
B Conduction of impulses
C Endocrine
D Protective
E Regenerative
An organ of the cardiovascular system is at microscopic preparations. One of its
layers is constructed of fibers that connected together. They are formed from cells that
are connected by intercalated discs. What is the organ of the cardiovascular system?
A Heart*
B Vein of muscular type
C Artery of muscular type
D Artery of elastic type
E Arterioles
The man is diagnosed an epithelial tumor of the pericardium. What epithelium is a
source of tumor development?
A Simple squamous *
B Pseudostratified epithelium
26
C Transitional
D Stratified epithelium keratinized
E Stratified epithelium non-keratinized
9
Arteries. Viens
Morphological examination revealed in histological specimen of biopsy material an
irregular-shaped vessel. Its middle membrane is formed by bundles of smooth
myocytes and layers of connective tissue. What type of vessel is it?
A Vein of muscular type*
B Artery uf muscular type
C Lymphatic vessel
D Venule
E Arteriole
Obliterating atherosclerosis causes changes in the vessels of the lower extermities. A
histological specimen of such a vessel evidently presents both internal and external
elastic membranes; middle membrane contains a lot of myocytes. What vessel is
affected in case of this disease?
A Artery of muscular type*
B Artery of elastic type
C Artery of mixed type
D Vein with strongly developed muscles
E Lymph node
A specimen of the pia mater shows a vessel with no middle membrane in its wall, its
outer membrane adheres to the surrounding tissues; the inner membrane is made up of
the basal membrane and endothelium. Specify this vessel:
A Fibrous vein*
B Muscular vein with weakly developed muscular elements
C Muscular artery
D Arteriola
E Mixed artery
At histological section of the vessel it is well expressed internal and external elastic
membranes and many myocytes in the tunica media. What vessel is there in the
section?
A. Mixed artery
B. Muscle artery *
C. Muscle vein
D. Elastic artery
E. Unmuscle vien
Walls of vessels have significant enough morphological disagreements in a structure
of an average environment. Than occurrence of specific features of a structure of this
environment in different vessels is predetermined?
A. Maintenance contents of hormones in blood
B. Influence of endocrine system
C. Regulation of the central nervous system
D. Inductive influence of autonomic nervous system
E. Haemodinamic conditions *
Arteries of the large calibre during a systole are stretched and and come back in an
initial condition during dyastole, providing stability of a blood pressure. What presence
of elements of the vessel’s wall it is possible to explain it?
A. Collagenic fibers
B. Muscle fibers
C. Reticular fibers
D. Elastic fibers *
E. Fibroblasts
27
At morphological research in section of biopsy the irregular-shaped vessel which
tunica media is formed by smooth myocytes and layers of a connective tissue is
determined. What is a vessel?
A. Muscle vein*
B. Muscle artery type
C. Lymphatic vessel
D. Venule
E. Аrteriola
At the preparation blood vessels are presented. The intima is formed by endothelium
and subendothelium, tunica media - bundles of smooth muscle cells, layers of loose
connective tissue. The tunica adventitia is highly developed and formed by loose
connective tissue and some smooth muscle cells. What is this vessel?
A Vein of muscular type *
B Artery of muscular type.
C Nonmuscular vein
D Artery of mixed type
E Artery of elastic type
At histological section there is shown a blood vessel. Its inner coat is composed by
endothelium, subendothelium and internal elastic membrane. The middle coat is
enriched with smooth myocytes. Such morphological characteristics are typical for the
following vessel:
A Muscular-type artery *
B Elastic-type artery
C Capillary
D Non-muscular vein
E Muscular-type vein
A vessel presents at histological section. Its wall consists of endothelium, basal
membrane and loose connective tissue. What type of vessel is it?
A Vein of non-muscular type *
B Artery
C Vein of muscular type
D Hemocapillary
E Lymphocapillary
A histological section of spleen there was identified a vessel with a wall consisting of
endothelium and subendothelial layer, median coat is absent, exterior membrane knit
together with connective tissue layers of spleen. What vessel is it?
A Vein of non-muscular type *
B Vein of muscular type
C Artery of muscular type
D Arteriole
E Capillary
At histological section presents an artery. One of the membranes of its wall has flat
cells lying on the basal membrane. What type of cells is it?
A Endothelium *
B Mesothelium
C Smooth myocytes
D Fibroblasts
E Macrophages
A specimen of pia mater includes a vessel whose wall doesn't have the tunica media,
the tunica externa is adherent to the surrounding tissues, the intima is composed of a
basement membrane and endothelium. What vessel is it?
A Nonmuscular vein *
B Muscular vein with underdeveloped muscular elements
28
C Muscular artery
D Arteriole
E Artery of mixed type
10
Microvascular
rate.
On electronogramm the capillary has fenestres in endothelium and continuous
basal lamina. Named type of the capillary.
A. Sinusoidal
B. Continuous
C. Fenestrated*
D. Typical
E. Atypical
For a capillary characteristic presence pores in endothelium and basal lamina. What
is type of this capillary?
A. Typical
B. Continuous
C. Fenestrated
D. Sinusoidal *
E. Lymphatic
In histological preparation there are vessels that look like flattened endothelial tubes do
not contain basal lamina and pericytes, endothelium of blood vessels is fixed to the
collagen fibers of connective tissue. What is this vessel?
A Lymph capillary *
B Haemocapillary
C Arteriolar
D Venules
E Arteriolar-venule anastomoses
In the preparation there is one of the vessels of microvascular rate, the tunica media is
formed by 1-2 layers of smooth muscle cells which are located singly and have a
spiral direction. The tunica adventitia is represented by a thin layer of loose connective
tissue. Specify the type of vessel.
A Arterioles*
B Venules.
C Capillary.
D Artery
E Arteriolar-venule anastomoses
11
Bone marrow.
Haemocytopoiesis.
A 46 year old patient was admitted to the hematological department. It was found that
he had disorder of granulocytopoesis and thrombocytogenesis processes. In what
organ does this pathological process take pace?
A Red bone marrow*
B Thymus
C Spleen
D Lymphatic ganglion
E Palatine tonsil
After a massive irradiation of mice in laboratory conditions there was a destruction
of haemopoietic cells. At what class of haemopoietic cells there is stimulation of
haemopoietins?
А. I class
В. II class *
С. III class
D. Blasts
Е. Defferentiated cells
On a preparation the big polyhedral cells, with invagination on a surface in which
are located erythroblasts is defined determined. What organ is submitted on a
29
preparation?
A. Thymus
B. Tonsils
C. Red bone marrow*
D. Spleen
E. Lymph nodes
On examination punctate of red bone marrow ther were found a significant decrease
in the number of megakaryocytes. What will be with blood cells?
A Decrease the number of platelets*
B Decrease the number of red blood cells
C Decrease the number of eosinophils
D Decrease the number of neutrophils
E Decrease the number of B-lymphocytes
In myeloid tissue punctate ther were found cells in which during the process of
differentiation pycnosis and removing the nucleus are happened. What kind of
hematopoiesis, characterized by morphological changes in the data.
A Erytrocytopoiesis *
B Trombocytopoiesis.
C Hranulocytopoiesis
D lymphocytopoiesis
E Monocytopoiesis
Granulocytes are found at histological examination of biopsies of red bone marrow.
What changes take place in the nucleus during differentiation of these cells?
A Segmentation*
B Poliployidizatsiya.
C Piknoz
D enucleation.
E Increasing the size.
An electronic microphotograph shows a macrophagic cell with erythrocytes at
different stages of differentiation located along its processes. This is the cell of the
following organ:
A Red bone marrow *
B Thymus
C Spleen
D Tonsil
E Lymph node
In the microspecimen of red bone marrow there were revealed multiple capillares
through the walls of which mature blood cells penetrated. What type of capillares is
it?
A Sinusoidal*
B Fenestrational
C Somatical
D Visceral
E Lymphatic
In course of an experiment a big number of stem cells of red bone marrow were in
some way destructed. Regeneration of which cell populations in the loose connective
tissue will be inhibited?
A Of macrophages *
B Of fibroblasts
C Of pigment cells
D Of lipocytes
E Of pericytes
30
12
Thymus. Palatine
Tonsils.
Medullary substance of a hemopoietic organ's lobule in a histological specimen is
lighter coloured and contains epithelial bodies. What organ are these morphological
properties typical for?
A Thymus*
B Lymph node
C Spleen
D Liver
E Kidney
At the newborn child it is revealed hypoplasia of thymus. What kind of haemopoiesis
will be broken?
A. Lymphopoiesis*
B. Neutrophilopoiesis
C. Erythropoiesis
D. Monocytopoiesis
E. Megakariocytopoiesis
On histological preparation the organ wich consists of lobules that are surrounded by
layers of connective tissue. At the periphery of lobules the number of cells is more
than in the center, lymph nodules are absent. Which organ is represented?
A Thymus*
B Red bone marrow
C Lymph node
D Spleen
E Tonsils
The antigen independent proliferation and differentiation of T lymphocytes are studied
in the child with impaired immune reactivity. Punctate of what organ was taken for
study?
A Thymus*.
B Spleen.
C Lymph nodes.
D Red bone marrow.
E Tonsils
Histological examination of a 40 y.o. man's thymus revealed decreased share of
parenchymatous elements, increased share of adipose and loose connective tissue,
and its enrichment with thymus bodies. The organ's mass was unchanged. What
phenomenon is it?
A Age involution*
B Accidental involution
C Hypotrophy
D Dystrophy
E Atrophy
The specimens present sections of haemopoetic and immunogenetic organs. Organ
has lymph tissue forming different structures (lymph nodes, lobules, bars). In what
organ does antigen-independent proliferation and differantiation take place?
A Thymus*
B Lymphatic nodes
C Spleen
D Hemolymph nodes
E Tonsil
13
Spleen. Lymph
nodes.
Examination of a patient who was exposed to the X-ray radiation revealed damage of
wight pulp of the spieen. What cells of white pulp undergo pathological changes?
A Lymphocytes*
B Neutrophilic leukocytes
C Basophilic leukocytes
31
D Monocytes
E Tissue basophils
At morphological research of a spleen activization of immunity reactions in an
organism has been determined. In what structures of the given organ begins secondary
proliferation of T-lymphocytes?
A. Mantile zone of white pulp
B. Red pulp
C. Periarterial zone of white pulp*
D. Marginal zone of white pulp
E. Central zone of white pulp
On preparation the organ is submitted, in reticular stroma which matures cells of
blood and lymph nodules are placed. What organ is submitted on preparation?
A. Tonsils
B. Тhymus
C. Spleen *
D. Lymph node
E. Red bone marrow
At the preparation the organ is represented wich covered connective tissue capsule
from which trabeculae away. In the organ you can distinguish cortex, which contains
lymphatic nodules, and the medulla with lymphoid cells. Which organ is
represented on the preparation?
A Lymph node*
B Thymus
C Spleen
D Red bone marrow
E Tonsils
At histological preparation the organ’s parenchyma is represented by lymphoid tissue
that forms the lymph nodules, the latter are diffusely and contain a central artery.
Which organ is represented on the preparation?
A Spleen*
B Tonsils
C Lymph node
D Thymus
E Red bone marrow
At histological preparation there is an organs which cells form three types of lymphatic
structure: lymphatic nodules, sinuses and medullary cord. Which organ is
represented?
A Lymph node*
B Spleen
C Thymus
D Tonsils
E Red bone marrow
The spherical formations of lymphocytes were found on micropreparations. In the
middle of formations - the central artery. Which organ is examined?
A Spleen. *
B Kidney.
C Thymus.
D Bone marrow.
E Lymph node.
A student examinated two histological preparations. Both of them had organs that had
lymph nodules. On the first preparation - only follicles, and on the second- follicles
contained eccentrically vessel. Determine what kind of organ?
32
A First - lymph node, the second - spleen*
B The first- red bone marrow, the second-spleen
C The first - thymus, the second - spleen
D The first -liver, the second-lymph node
E The first - liver, the second-spleen
In a histological specimen parenchyma of an organ is represented by lymphoid tissue
that forms lymph nodes; the latter are arranged in a diffuse manner and enclose a
central artery. What anatomic formation has such morphological structure?
A Spleen *
B Tonsil
C Lymph node
D Thymus
E Red bone marrow
A specimen shows an organ covered with the connective tissue capsule with trabeculae
radiating inward the organ. There is also cortex containing some lymph nodules, and
medullary cords made of lymphoid cells. What organ is under study?
A Lymph node *
B Thymus
C Spleen
D Red bone marrow
E Tonsils
14
System of the
immunity
defence.
A patient has a skin defect as a result of an extensive burn. In order to mask this defect
the surgeons transplanted a skin flap from other body part of this patient. What
type of transplantation is it?
A Autotransplantation*
B Explantation
C Allotransplantation
D Xenotransplantation
E Homotransplantation
Blood analysis of a 16-year-old girl suffering from the autoimmune inflammation of
thyroid gland revealed multiple plasmatic cells. Such increase in plasmocyte number
is caused by proliferation and differentiation of the following blood cells:
A B-lymphocytes*
B T-helpers
C Tissue basophils
D T-killers
E T-supressors
At older people’s frequency of occurrence of tumors rises. One of principal causes
of it:
A. Decreasing of intensity of antibodies’ formation
B. Increasing of frequency of infringements mitosis
C. Decreasing of activity of cellular immunity*
D. Increasing of activity of cellular immunity
E. Increasing of activity of antibodies’ formation
To the patient with the big burns have made change of a donor’s skin. But for 8 day
transplantant’s color has changed and for 11 day it started to be torn away. What cells
accepted in it participation?
А. T-lymphocytes*
B. Erythrocytes
С. B-lymphocytes
D. Acidophills
E. Basophills
33
A patient 30 years was diagnosed with skin tumor. What epidermal cells take part in
the immune response?
A T-lymphocytes*
B Keratinocytes
C Keratinocytes and Merkel cells
D Merkel cells
E Cells of stratum spinosum
Burn wound was closed pig skin (heterotransplantation). Name the effector cells with
reject the transplant (pig skin).
A T-killer*
B T-helper
C T-suppressor
D B lymphocytes
E Natural killer
Antibodies are produced when antigen enters the body repeatedly. With the
function of what immune cells this phenomenon linked?
A Memory lymphocytes*
B T - killer
C T - suppressor
D Macrophages
E Dendritic cells
In girl’s blood 16 years old which suffers autoimmune inflammation of thyroid
gland, numerous plasma cells are found out. With proliferation and differentiation of
what blood cells connects increase in quantity amount plasma cells?
A. T-lymphocytes
B. T-helpers
C. Mast cells
D. Т-killers
E. B-lymphocytes *
15
Endocrine system.
Hypothalamus.
Epiphysis.
A 35 year old patient complains about permanent thirst, poor appetite. He drinks 9 l
water per day. Daily diuresis is increased, urine is colourless, and its relative
density is 1,005. The most probable cause of this pathology development is damege
of:
A Hypothalamic nuclei*
B Epithelium of renal tubuli
C Adenohypophysis
D Epiphysis
E Basal membrane of glomerular capillaries
Woman 40 years old have powerless childbirth, caused by weakness of contractile
abilities of myometrium. What hormonal preparation needs to be entered to help her?
A. Аldosterone
B. Cortisol
C. Dexamethasone
D. Оxitocine *
E. Prednisolone
16
Hypophysis.
Hypothalamohypophyseal
system.
The aim of the morphological study was to investigate an endocrine gland with
parenchyma consisting of epithelium and neural tissue. In the epithelial trabeculae
the study revealed two types of cells: chromophile and chromophobe. Identify this
organ:
A Pituitary gland*
B Adrenal gland
C Hypothalamus
D Thyroid gland
34
E Parathyroid gland
The patient of very high growth comes to the doctor. He has long thick fingers of
hands, the big bottom jaw and the drooped lower lip. Increased secretion of what
hormone can be suspected?
A. Somatotrophin*
B. Thyrotroph
C. Gonadotroph
D. Vasopressin
E. Oxitocin
Growth of the child of 10 years reaches achieves 178 sm, weight - 64 kg. What
endocrine gland it is connected to infringement of activity?
A. Reproductive
B. Suprarenal
C. Thyroid
D. Hypophysis*
E. Parathyroid
A month later after childbirth the woman of 25 years addressed to the doctor with
complaints to decrease reduction in quantity amount of milk. What reduction of
secretion of hormone has led to such condition?
A. Thyrotrophin
B. Insuline
C. Prolactine*
D. Gonadotrophin
E. Somatotrophin
Student investigated endocrine gland. Its parenchyma consists of epithelium and
neural tissue. Epithelial trabecules have two types of cells: chromophilic and
chromophobic. Identify this organ:
A Hypophysis *
B Adrenal glands
C Hypothalamus
D Thyroid gland
E Parathyroid gland
17
Thyroid gland.
Parathyroid
glands.
A 9 y.o. boy was admitted to the endocrinological department. This boy has already
had several fractures of his extremities due to bone brittleness. The function of the
following endocrinal glands (gland) is disturbed:
A Parathyroid*
B Thyroid
C Thymus
D Adrenal
E Epiphysis
Parodontitis is treated with calcium preparations and hormones that stimulate tooth
mineralization and inhibits tissue resorption. What hormone is it?
A Calcitonin*
B Parathormone
C Adrenalin
D Aldosterone
E Thyroxine
Clinical examination of a female patient revealed reduction of basal metabolism by
40%, gain in body mass, drop of body temperature, face puffiness, sexual disfunctions,
inertness and apathy, lowered intelligence. These symptoms are caused by dysfunction
of the following endocrine gland:
A Hypofunction of thyroid gland*
35
B Hypofunction of parathyroid glands
C Hypophysis hyperfunction
D Epiphysis hypofunction
E Hyperfunction of thyroid gland
After a surgical procedure an experimental animal died from intense convulsions.
What endocrinal glands were extracted?
A Parathyroid*
B Thyroid
C Adrenal
D Ovaries
E Testicles
A 5-month-old boy was hospitalized for tonic convulsions. He has a life-time history
of this disease. Examination revealed coarse hair, thinned and fragile nails, pale and
dry skin. In blood: calcium - 1,5 millimole/l, phosphor - 1,9 millimole/l. These changes
are associated with:
A Hypoparathyroidism*
B Hyperparathyroidism
C Hyperaldosteronism
D Hypoaldosteronism
E Hypothyroidism
The woman of 53 years, growth of 163 sm, weight of 93 kg, fat constitution, the face
with edema, inactive, apathetic. Infringement of functions what endocrine gland
causes a condition of the patient?
A. Suprarenal
B. Gonades
C. Parathyroid
D. Thyroid*
E. Hypophysis
On histopreparation it is submitted parenchymal organ. Parenchyma consists of the
many follicles which wall is formed by a layer of cuboidal cells. In a gleam of
follicles collects colloid. To what organ such morphological attributes are
characteristic?
A. Thyroid gland*
B. Ovaries
C. Lymph node
D. Hypophysis
E. Suprarenal gland
At the patient for the third day after total thyroidectomia muscle spasmes have
appeared. What medical products should be appointed this patient?
A. Preparates of calcium*
B. Antispasmatic drags
C. Sedative drags
D. Antibiotics
E. Preparates of potassium
Kidneys of a man under examination show increased resorbtion of calcium ions and
decreased resorbtion of phosphate ions. What hormone causes this phenomenon?
A Parathormone *
B Thyrocalcitonin
C Hormonal form D 3
D Aldosterone
E Vasopressin
18
Suprarenal gland.
In a histological specimen of adrenal cortex there are petite polygonal cells that form
36
roundish clusters and contain some lipidic inclusions. What part of adrenal is
presented in this histological specimen?
A Glomerular zone*
B Intermedial zone
C Fasciolar zone
D Reticular zone
EA patient ill with adenoma of glomerular zone of adrenal cortex (Conn's disease)
has arterial hypertension, convulsions, and polyuria. What is the main factor in the
pathogenesis of these disturbances?
A Aldosterone hypersecretion*
B Aldosterone hyposecretion
C Catecholamines hypersecretion
D Glycocorticoids hypersecretion
E Glycocorticoids hyposecretion
Microscopic examination of a parenchymatous organ revealed that its epithelial cords
formed glomerular, fascicular and reticular zones. The central part of the organ was
presented by accumulations of chromaffin cells. Specify this organ:
A Adrenal gland*
B Thyroid gland
C Epiphysis
D Liver
E Hypophysis
A patient with Itsenko-Cushing syndrome has persistent hyperglycemia and
glycosuria, hypertension, osteoporosis, obesity. Increased synthesis and hypersecretion
of the following hormone will be observed in this case:
A Cortisol*
B Adrenaline
C Glucagon
D Thyroxin
E Aldosterone
At the man of 42 years which long time was in a condition of stress, in urine the
contents of 17-ketosteroids is considerably increased, that first of all testifies to
increase of secretion:
A. Epinephrine *
B. Estradiol
C. Kortizol
D. Norepinephrine
E. Aldosterone
Organ’s parenchyma forms dense glomerulus, formed from endocrinocytes. To
what organ the given structure is characteristic?
A. Lymph node
B. Spleen
C. Kidney
D. Suprarenal gland*
E. Pancreas
A patient has been given high doses of hydrocortisone for a long time. This caused
atrophy of one of the adrenal cortex zones. Which zone is it?
A Fascial *
B Glomerular
C Reticular
D Glomerular and reticular
E-
37
19
20
Control
of
mastering of the
submodule 3.
General
description
of
organs of the
digestive system.
Development of
face and organs of
oral cavity
In course of embryogenesis maxillary and mandibular processes grow together
with a delay. What development anomalies should be expected in this case?
A Macrostomia*
B Microstomia
C Cleft palate
D Gothic palate
E Cleft of superior lip
The constant nasal regurgitation of milk from a nose has been detected at
newborn during the first feeding. What anomaly of development does this
symptom testify to?
A. Bronhiogenic fistula
B. Makrostomija
C. Mikrostomija
D. Hiatus of palate*
E. Hiatus of lips
At the baby during the certificate acts of swallowing nasal regurgitation of milk
from a nose. At survey of the child the surgeon has found out congenital defect, socalled “the wolf mouth”. Specify what anomalies of development result in occurrence
of the given pathology.
A. Defect between mandibular processes
B. Infringement of accretion of average of nasal and maxillar processes
C. Defect between palatal processes *
D. Defect between lateral lingual tuberculums
E. Infringement of development of the frontal processes
At the newborn child defect of formation of the human - a medial crevice of the
upper lip is determined. What infringement of growth of human’s a germ has taken
place in this case?
A. Medial nasal processes*
B. Lateral nasal and maxillar processes
C. Maxillar and mandibular processes
D. Palatal processes
E. Lateral lingual swellings
21
Description of the The stomatologist in cabinet study has given the task to the intern to survey at the
oral mucosa. Lip. patient sites of oral mucousa, which keratinazed. What it is a site?
Cheek. Gums.
A. Ventral surface of the tongue and palatine tonsils
B. Hard palate, ventral surface of the tongue, gingiva
C. Cheeks, lower lip, tongue
D. Palate and a diaphragm of the mouth
E. Hard palate, marginal zone of gingiva and intermediate part of cheeks *
The student has remembered, that epithelia of oral mucosa non-keratinazed. On a
histological preparation of the cheek he has seen, that from both its parties sides
keratinized epithelia. What the student should remember?
A. Mandibular zone of a cheek can keratinized
B. Maxillar zone of a cheek can keratinized
C. Intermediate zone of a cheek can keratinized*
D. Place of parotid duct opened can keratinized
E. Marginal zone of a cheek can keratinized
On a histological preparation oral cavity’s organ which will consist of three parts, skin, intermediate and mucous is seen. The basis is formed striated muscle tissue.
38
22
What it for body?
A. Cheek
B. Gingiva
C. Hard palate
D. Soft palate
E. Lip*
Development and A histological specimen of an oral cavity organ demonstrates that the organ's anterior
structure
of surface is lined with squamous nonkeratinized epithelium, and its posterior surface tongue. Hard and with pseudostratified ciliated epithelium. What organ is it?
soft palate
A Soft palate*
B Gingiva
C Hard palate
D Lip
E Cheek
A child damaged the lateral surface of his tongue. What lingual papillas are most
likely to be damged?
A Foliate*
B Conic
C Vallate
D Filiform
E Fungiform
There is a specimen of soft palate where both oral and nasal surfaces can be seen. It
was revealed that oral cavity had damaged epithelium. What epithelium is damaged?
A Stratified squamous nonrtratinized*
B Stratified cuboidal nonrtratinized
C Stratified cilindrical nonrtratinized
D Stratified squamous keratinized
E Pseudostratified ciliated epithelium
On a histological preparation of oral cavity’s organ it is visible 4 zones: adipouse,
glandular, sutural and marginal. What for organ on a preparation?
A. Soft palate
B. Gingiva
C. Hard palate *
D. Lip
E. Cheek
On a histological preparation of oral cavity’s organ it is visible, that the one surface is
covered by non-keratinized epithelia, and the second surface – pseudostratified
epithelia. What it for body?
A. Cheek
B. Gingiva
C. Hard palate
D. Soft palate *
E. Lip
At the sick child the white strike in tongue is observed. Due to what papilla there is a
given phenomenon?
A. Circumvallate
B. Foliate
C. Conic
D. Fungiform
E. Filiform *
As a result of the chemical burn of the lateral surface of tongue at 5 years child in this
site there is no taste sensitivity that is connected to destruction of taste touch cells.
What structures of the cell take part in perception recognition of taste?
39
A. Microvilli on apical surfaces *
B. Lateral surface
C. Microvilli on basal surfaces
D. Nucleus
E. Basal lamina
23
Large and small At micropreparation of the submandibular salivary gland there are shown some
salivary glands
basket-shaped cells concentrated around the acini and excretory ducts. These cells
surround bases of the serous cells and are called myoepitheliocytes. These cells relate
to the following tissue:
A Muscle tissue *
B Epithelial tissue
C Nervous tissue
D Special connective tissue
E Loose fibrous connective tissue
A histological specimen of a mandibular gland shows an exctretory duct. Mucous
membrane of the duct is lined with cuboid epithelium whose cells have weakly
developed organellas. What excretory duct is it?
A Intercalated*
B Striated
C Interlobular
D Common excretory
EA microspecimen of parotid gland presents secretory acini with serous cells that
synthesize mostly enzymes. According to the chemical composition classification, the
parotid gland relates to the following glands:
A Serous*
B Mucous
C Seromucous
D Enzymatic
EExamination of a microspecimen made of an unknown organ revealed some acini that
contained 10-15 cone cells with basophilic cytoplasm, round nucleus and well
developed granular endoplasmic reticulum. An acinus is surrounded by a basal
membrane with myoepithelial cells localized in its splitting. What organ is the slice
made of?
A Parotid gland*
B Pancreas
C Lungs
D Sublingual gland
E Liver
At an acute inflammation of parotid gland processes of secretion and allocation are
broken. What cells suffer thus?
A. Serous, mucous
B. Albuminous, serous, mucous
C. Serous, cells with basal striation, stellate cells
D. Serous, myoepithelial cells *
E. Bordered epitheliocytes, cells with basal striation
At chronic inflammatory processes of salivary glands damage of ductal epithelia is
observed. What epithelia will be damaged injured thus in striated ducts of major
salivary glands?
A. Cuboidal epithelia with basal striation
B. Cilindrical epithelia with basal striation *
40
C. Squamous epithelia with basal striation
D. Two-layer epithelia with basal striation
E. Multilayered cuboidal
Parotid gland has the end-pieces formed by serocytes. What organelles of these cells
provide synthesis and secretion of saliva’s components?
A. Smooth endoplasmic reticulum
B. Golgy complex
C. Rough endoplasmic reticulum *
D. Mitochondria
E. Lysosomes
In structure of end-pieces of salivary glands, except for secretory exocrinocytes,
basket-shape cells which are placed outside of the secretory layer are determined and
promote receipt of products of secretion to ductal system. What special organelles it is
possible to determine in cytoplasm of basket-shape cells?
A. Villi
B. Acrosome
C. Neurofibrilles
D. Myofilaments*
E. Flagella
24
Tooth structure.
Before teeth come out first on their roots appears a solid tissue that looks like woven
bone. What tissue is it?
A Cement*
B Dentin
C Enamel
D Loose fibrous connective tissue
E Dense fibrous connective tissue
A histological specimen presenting a tooth slice shows that the intercellular dentin
substance contains collagen fibers being tangential to the dentinoenamel junction and
perpendicular to the dentinal tubules (Ebner's fibers). This dentin layer is called:
A Nearpulp dentin*
B Mantle dentin
C Granular layer
D Interglobular dentin
E Secondary dentin
Histological examination of trasverse enamel slice revealed linear banding in form of
concentric circles that is pointing at an angle to the dentinoenamel junction. Name
these structures:
A Retsius' lines*
B Hunter-Schreger's lines
C Enamel plates
D Enamel fascicles
E Enamel spindles
It was revealed that a 42 y.o. patient suffering from paradontosis had roundish
calcified formations 2-3 mm in diameter in the coronal pulp. Name these structures:
A Denticles*
B Interglobular spaces
C Sclerotic dentin
D Dead dentin
E Intertubular dentin
Examination of a tooth slice of a 42 y.o. man revealed on the dentinal-enamel
border some solid inear fusiform structures as long as 1/3 of enamel depth. What
structures were revealed?
41
A Enamel spindles*
B Denticles
C Enamel fascicles
D "Dead" tracts
E Carious damage
A histological specimen presents a developed tooth that has a coating resistant to
acids, but it can be found only on the lateral surfaces of the tooth. What coating is
meant?
A Cuticle*
B Dentine
C Enamel pellicle
D Enamel
E Cement
In a peripheral zone of the pulp for the certain reasons activity of cells is temporarily
delayed. What tissue of the tooth appears under threat of infringement of physiological
regeneration?
A. Enamelum
B. Dentine *
C. Pulp
D. Cellular cementum
E. Ancellular cementum
In two preparations of a tooth histological it is determined: in one - uncellular
cementum, in the second - cellular. The second preparation is made of what site of a
tooth?
A. From the apex of the root *
B. From the cervix of the tooth
C. From the bone
D. From crown of the tooth
E. From a site of border between crown and the root
In nearpulp dentine of the tooth of the adult person sites with the disorder
accommodation of dentinal tubules and collagenic fibers are determined. Named the
given kind of dentine.
A. Secondary (regular).
B. Primary
C. Tertiary (irregular) *
D. Sclerotic
E. Dead ways
The child complains of a toothache. The doctor - stomatologist ascertained caries
damage of enamel. What quantity amount of mineral substances is reduced in a site of
caries damages?
A. Phosphorus, fluorine, calcium *
B. Sodium, calcium, kalium
C. Кalium, phosphorus, fluorine
D. Magnesium, fluorine, calcium
E. Phosphorus, magnesium, kalium
At studying of tooth’s slice of the human of 40 years in a site of the root the
transparent zone in which are have now dental tubules is determined. What age
changes of dentine are observed on slice?
A. Formation of sclerotic dentine*
B. Formation of denticles
C. Condensation of dentine
D. Formation of dead ways
E. Caries damage
42
25
Tooth
development.
During the embryogenesis of oral cavity the development of dental enamel was
disturbed. What source of dental development was damaged?
A Epithelium*
B Mesenchyma
C Mesoderma
D Dental saccule
E Dental papilla
During the tooth development the enamel organ has prismatic cells with hexagonal
intersection; the nucleus is situated in the central part of the cell. What cells are
meant?
A Preameloblasts*
B Exterior enameloblasts
C Cambial cells
D Enamel pulp cells
E Preodontoblasts
A histological specimen of mandible of an embryo shows a tooth germ with the dental
papilla made up of small stellate basophilic cells. What tissue forms this part of the
tooth germ?
A Mesenchyma*
B Epithelial
C Reticular
D Cartilaginous
E Osseous
Histological study of an extirpated pulp revealed some cylindrical cells in its
peripheral layer. What are these cells called?
A Odontoblasts*
B Fibroblasts
C Monocytes
D Ameloblasts
E Myofibroblasts
As a result of the certain reasons on 5 month after a birth of the child process of
destruction of Hertvig’s vagina around of the tooth’s root is broken. Development of
what tooth’s tissue is prevented by this circumstance?
A. Pulp
B. Dental papilla
C. Dental sac
D. Cementum*
E. Dentine
At examination, answering a question on development of hard and soft tissues of the
tooth, the student has made a mistake, when has told, that enamel is formed of
mesenchyma’s cells. What should be a right answer?
A. From an internal layer of enamel organ *
B. From cells of pulp of the enamel organ
C. From external layer of enamel organ
D. From external layer of dental sac
E. From external cells of dental papilla
During formation of peripheral dentine in the tooth there was an infringement of
secretory activity of odontoblasts. Define determine, what formation of fibers will
change?
A. Elastic
B. Reticular
C. Radial collagenic fibers *
43
D. Tangential collagenic fibers
E. Nervous
On a histological preparation of the developmental jaw it is determined dentine which
is at the stage of formation. Collagenic fibers which synthesize odontoblasts thin and
are focused perpendicularly to dentine tubules. What fibers are formed in dentine?
A. Parallel
B. Radial
C. Tangential*
D. Circular
E. Angular
On a histological preparation the cut of the jaw of the human’s germ of 2-nd month of
development which contains damaged injured of dental epithelial organ is submitted.
What histological part of the tooth will not develop?
A. Enamel *
B. Pulp
C. Cementum
D. Periodontium
E. Dentine
An anomaly of development of enamel has been found out. To what damage of
structural components of the dental germs it is connected?
A. External epithelia of the enamel organ
B. Internal epithelia of the enamel organ*
C. Pulp of the enamel organ
D. Dental sac
E. Dental pulp
In process of embryonic developments there was a damage superficial mesenchymal
cells of dental papilla. It can result in infringement of formation of what structure of a
tooth?
A. Dentine *
B. Enamel
C. Cementum
D. Periodontium
E. Cuticle
In the process of the tooth’s morphogenesis there was a damage of internal cells of
dental sac. What formation of structures of the tooth will be broken?
A. Pulp
B. Enamel
C. Dentine
D. Cementum *
E. Periodontium
An insufficient development of a pulp of the tooth was revealed. What embryonic
source has tested damage?
A. Mesenchyma *
B. Ectoderm
C. Endoderm
D. Oral epithelia
E. Nervous tube
On a histological preparation of sagital section of developmental jaw of a 3,5-month's
human fetus it is observed epithelial enamel organ surrounded by compactly located
mesenchymal cells. How it refers to mesenchymal formation?
A. Dental papilla
B. Dental sac *
44
C. External enamel cells
D. Internal enamel cells
E. Pulp of enamel body
The child of 11 years in an x-ray picture does not have lateral teeth (adentia). It is
connected with:
A. Infringement of formation of dental papilla
B. Infringement of formation of dental sac
C. Infringement of formation of enamel organ *
D. Infringement of formation of dentine
E. Infringement of formation of cementum
In embryogenesis of oral cavity there was an infringement of development of enamel
of the teeth. What source of teeth’s development has been broken?
A. Mesoderm
B. Mesenchyma
C. Epithelia *
D. Dental sac
E. Dental papilla
During development of the tooth in structure of enamel organ there are cells with
prismatic shape which have six-angular section; the nucleus is located in the central
part of a cell. What it for cells?
A. Preameloblasts *
B. External ameloblasts
C. Stem cells
D. Cells of the pulp of enamel organ
E. Preodontoblasts
After formation of dentine process of inversion in cells - moving of the nucleus and
organelles begins. What cells this process concerns?
A. Odontoblasts
B. Ameloblasts*
C. Preodontoblasts
D. Cementoblasts
E. Cementocytes
At microscopic research of biopsy it was revealed infringement of development of the
periodontum. What source of development of the tooth is broken?
A. Dental papilla
B. Dental sac*
C. Dental plate
D. Preameloblasts
E. Ameloblasts
Experimentally at the germ in a dental germ the external layer of dental sac has been
destroyed. Specify, what structure of a tooth will not have the subsequent
development?
A. Pulp
B. Enamel
C. Dentine
D. Cementum
E. Periodontium*
At the second stage of development of the tooth in enamel organ and dental papilla
polyhedral cells are formed. To what attributes it is possible to determine a bookmark
of the pulp of the tooth on preparations, taking into account, what it has mesenchyme
origin?
A. On presence of blood vessels*
45
B. Under the shape of nucleus
C. By quantity amount of shoots
D. On basophilia of cytoplasma
E. It is impossible to distinguish
The beginning of tooth’s eruption there is a tertiary mineralization of enamel. From
what sources mineral substances mainly act?
A. Gingiva
B. Dentine
C. Periodontum
D. Saliva*
E. Pulp
Process of maturing of enamel (a secondary mineralization) is provided with
structurally functional transformation of ameloblasts from secretory in maturing.
What morphological attributes allow determining on a preparation maturing
ameloblasts?
A. Absence of Toms’ processes*
B. Presence of apical border
C. Absence of apical border
D. Basophilic cytoplasm
E. Destruction of a nucleus
26
Periodont.
Eruptions and
changing of teeth
During examination of a child's oral cavity a pediatrician found 8 incisors. The child's
development corresponds to his age. How old is the child?
A 10-12 months*
B 6-7 months
C 7-8 months
D 12-15 months
E 16-20 months
During examination of a child's oral cavity a pediatrician established presence of
inferior medial incisors. The child's development is normal. How old is the child?
A 6-7 months*
B 8-9 months
C 10-12 months
D 13-14 months
EThe mistake of the student in his answer to the question on structure of parodontum
was that it has named among other tissues the cartilage and dental sac. What should be
a right answer?
A. Periodontium, alveolar bone and cementum *
B. Pulp, root, periodontium
C. Periodontium, cementum, alveolus’s periosseum
D. Cervixof the tooth, periodontium, gingiva
E. Gingiva, gingival papilla, alveolus of the tooth
Before eruption of constant teeth there is the destruction of the supporting tissues of
the lactic tooth. What biological process provides fast destruction of the connective
tissue of the periodontum without development of an inflammation?
A. Mitosis
B. Necrosis
C. Meiosis
D. Apoptosis*
E. Fibrinolysis
27
Esophagus.
Stomach.
A patient underwent gastroscopy that revealed insufficient amount of mucus
covering the mucosa. This phenomenon is caused by the dysfunction of the following
46
cells of stomach wall:
A Cells of prysmatic glandular epithelium*
B Parietal cells of gastric glands
C Chief-cells of gastric glands
D Cervical cells of gastric glands
E Endocrinocytes
3 years ago a 52 y.o. man underwent an operation for stomach extraction. Results of
blood analysis: erythrocytes - 2,0x1012/l, Hb- 85 g/l, colour index - 1,27. These
changes were caused by disturbed assimilation of the folowing vitamin:
A B12*
B B6
CC
DP
EA
Examination of a patient, suffering from atrophic gastritis, revealed megaloblastic
anemia. The anemia is likely to be caused by the deficiency of the following
substance:
A Gastromucoproteid*
B Vitamin B6
C Vitamin B1
D Iron
E Erythropoietins
The man of 35 years with a stomach ulcer the resection of antral department of
stomach is made. Which secretion of gastrointestinal hormone as a result of
operation will be broken most of all?
A. Gastrin*
B. Secretin
C. Neurotenzin
D. Hystamine
E. Holecystokinin
At the patient at gastroscopy the insufficient quantity amount of mucus on a surface
of mucosa is revealed. To what infringement of function of stomach’s cells it is
connected?
A. Parietal cells
B. Columnar cells*
C. Endocrinocytes
D. Cervical cells
E. Chief cells
On histologic preparation the cross-section of alimentary tubular organ’s wall which
mucous shall is covered by nonkeratinazed epithelia is submitted. What is the
organ?
A. Uterus
B. Duodenun
C. Small intestine
D. Esophagus *
E. Appendix
On the fourth week of embryonic developments occurs physiological atresia of
esophagus. Up to the end of the eighth week it again becomes passable. What
biological process provides recanalisation of esophagus at human embryo?
A. Descvamation
B. Меiosis
C. Necrosis
D. Apoptosis*
47
E. Mitosis
During histological examination of the stomach it was found out a significant
reduction or complete absence of parietal cells in the glands. Mucose membrane of
what part of the stomach was studied?
A Pyloric part*
B Fundus of stomach
C Cardia
D Body of stomach
E When the pH level of the stomach lumen decreases to less than 3, the antrum of the
stomach releases peptide that acts in paracrine fashion to inhibit gastrin release.
This peptide is:
A GIF *
B Acetylcholine
C Gastrin-releasing peptide (GRP)
D Somatostatin
E Vasoactive intestinal peptide (VIP)
Examination of a 43 y.o. patient revealed that his stomach has difficulties with
digestion of protein food. Gastric juice analysis revealed low acidity. Function of
which gastric cells is disturbed in this case?
A Parietal exocrinocytes *
B Main exocrinocytes
C Mucous cells (mucocytes)
D Endocrinous cells
E Cervical mucocytes
An electron microphotography of a fragment of proper gastric gland shows a big
irregular round-shaped cell. There are a lot of intracellular tubules and
mitochondria in the cytoplasm. Specify this cell:
A Parietal cell *
B Principal cell
C Undifferentiated cell
D Mucous cell
E Endocrine cell
A patient ill with chronic gastritis went for endogastric pH-metry that allowed
revealing decreased acidity of gastric juice. It is indicative of diminished function of
the following cells:
A Parietal exocrinocytes *
B Chief exocrinocytes
C Endocrinocytes
D Cervical cells
E Accessory cells
28
Small and large
intestine.
What deficiency of enzyme is the reason of incomplete digestion of fats in a
gastroenteric path and increases in quantity amount of neutral fat in feces more
often?
A. Lipase*
B. Enterokinase
C. Lactase
D. Dipepttidase
E. Secretine
On histologic section of small intestine’s wall at the crypts’ cells located by groups
are found, in apical parts contain big acidophilic secretory granules; cytoplasm is
basophilic. What are these cells?
48
A. Columnar cells
B. Paneth cells *
C. Endocrinocytes
D. Goblet cells
E. Enterocytes
At the patient with chronic enterocolitis (the inflammation of intestines) is revealed
infringement of digestion and absorption digestive products in thin gut as a result
of insufficient quantity amount in intestinal juice dipeptidases. In what cells synthesis
of these enzymes is broken?
A. Goblet cells
B. Columar cells without brush border
C. Paneth cells *
D. Stem cells
E. Enterocytes with a brush border
The patient with thyreotoxicosis complains of diarrhea, heavy feeling in stomach. At
examination - feces without pathological changes. At radiological research definitely
acceleration of passage of baric masses along intestines. The hypertonus of what
shall of alimentary tube is the reason of the given condition?
A. Serous
B. Mucous
C. Adventitial
D. Submucous
E. Muscular*
29
Liver.
On histological preparation parenchyma of organ it is submitted by lobes which have
the 6-angular prisms shape and consist from trabeculas between sinusoidal
capillaries which radiality converge to the central vein. What anatomic organ has the
given morphological structure?
A. Тhymus
B. Pancreas
C. Liver *
D. Spleen
E. Lymph node
There is large number of carbohydrates in the human diet. What structures are
detected while in the cytoplasm of hepatocytes?
A Glycogen granules*
B Droplets of fat.
C One big fat drop.
D Increase the number of free ribosomes.
E The inclusion of lipofuscin.
A viral infection has damaged cells that form walls of bile capillaries. This
stimulated conditions for inflow of bile into the blood of sinusoidal capillaries. What
cells are damaged?
A Hepatocytes *
B Kupffer's cells
C Ito cells
D Pit-cells
E Endotheliocytes
30
Pancreas.
The patient since 14 years old has diabetes. What endocrine cells of pancreatic islands
don’t function?
A. D - cells
B. A - cells
C. D1-cells
D. B - cells *
49
E. РР - cells
In patients after acute pancreatitis is determined massive damage of acinar cells. By
which cells will go their recovery?
A Intercalated duct cells*
B Cells of islets of Langerhans
C Cells of interlobular duct
D Cells of gland’s stroma
E Endothelium of blood vessels
Endocrinologist diagnosed in patient disorders of the endocrine function of the
pancreas, resulting descending of the hormone glucagon in the blood. The function of
this gland cells broken in this case?
A A-cells of the islets of Langerhans*
B B-cells in the islets of Langerhans
C D-cells of islets of Langerhans
D D1-cells of islets of Langerhans
E PP-cells of islets Langerhans
31
Respiratory
system.
A patient with an acute rhinitis has hyperemia and excessive mucus formation in
nasal cavity. What epithelial cells of mucous membrane have the intensified activity?
A Goblet cells*
B Ciliated cells
C Microvillous cells
D Basal cells
E Endocrine cells
Premature infants have syndrom of respiratory failure. Failure of what aerohematic
barriere component underlies this pathology?
A Surfactant*
B Capillary endothelium
C Basal membrane of endothelium
D Basal membrane of alveolocytes
E Alveolocytes
At the patient with a dry pleuritis noise of friction of pleura is listened. At what defeat
of type of epithelia this symptom is marked?
A. Compound
B. Simple squamous*
C. Transitional
D. Cuboidal
E. Cilindrical
In a histologic preparation the organ which wall consists of mucosa, submucosa, fibrocartilage and аdventitial shalls is submitted. Epithelia - pseudostratified, lamina
muscularis is absent, in submucous – mixed glands, hyaline cartilage forms the openended rings. What organ has this morphological attributes?
A. Trachea*
B. Terminal bronchiole
C. Segmental bronchi
D. Alveola
E. Larynx
In an electronic microphoto the cells of alveoles are founded. They are the part of aerohaematic barrier. What are these cells?
A. Secretory alveolocytes
B. Respiratory alveolocytes *
C. Alveolar macrophages
D. Endocrinocytes
50
E. Microphages
It is known, that the important component of aero-haematic barrier is surfactant which
warns closed alveoli during breathe in. By what cells of alveoli are synthesized
phospholipids which constructed these membranes?
A. Alveolocytes ІІ type *
B. Respiratory cells
C. Goblet cells
D. Alveolar macrophages
E. Endotheliocytes of capillaries
In patient with laryngeal diphtheria doctor founded membranous coats on mucosa, that
strong connected with epithelia. To what types of epithelia, marked below, the
membranous coats fixed strong?
A. Compound keratinazed
B. Compound non-keratinazed *
C. Pseudostatified
D. Simple sqamous
E. Simple cilindrical
In histologic preparation of the organ which wall has mucous, submucous, fibrouscartilage and adventitial shall is submitted. Epithelia - pseudostratified. In submucous
– end-pieces of the mixed glands. Hyaline cartilage forms large plates. What organ
has the given morphological attributes?
A. Large bronchi *
B. Trachea
C. Small bronchi
D. Larynx
E. Middle bronchi
In an electronic microphoto of biopsy structures into which structure enters surfactant,
alveolocytes I type, basal lamina and fenestrated epithelia of capillaries are submitted.
What histo-haematic barrier in human body content from these structures?
A. Haemato-encefalic
B. Haemato-thymic
C. Aero-haematic *
D. Uro-haematic
E. Haemato-testicularis
The patient has acted arrived in branch with attack of asthma which is caused by
spasm of smooth muscles of respiratory ways. Name departments of respiratory
ways to which basically it is connected this attack.
A. Bronchi of small caliber*
B. Bronchi of middle caliber
C. Bronchi of large caliber
D. Terminal bronchioles
E. Respiratory departments
Lung of premature infant is presented on electronic photomicrography of biopsy
material. Collapse of the alveolar wall caused by the deficiency of surfactant was
revealed. Disfunction of what cells of the alveolar wall caused it?
A Alveocytes type II *
B Alveocytes type I
C Alveolar macrophages
D Secretory cells
E Fibroblasts
A patient was admitted to the hospital with an asphyxia attack provoked by a spasm of
smooth muscles of the respiratory tracts. This attack was mainly caused by
51
alterations in the following parts of the airways:
A Small bronchi *
B Median bronchi
C Large bronchi
D Terminal bronchioles
E Respiratory part
At electronic microphotography of pulmonary alveole's wall a big cell presents. Its
cytoplasm has a lot of mitochondria, developed Golgi apparatus, osmiophil lamellated
corpuscles. What is the main function of this cell?
A It produces surfactant *
B It is a component of blood-air barrier
C It warms the air
D It purifies the air
E It absorbs microorganisms
A pathological process in bronchi caused desquamation of epithelium. What cells will
regenerate bronchial epithelium?
A Basal *
B Intercalary
C Ciliate
D Endocrinal
E Goblet
Alveolar space of the acinus was invaded by some bacteria which interacted with the
surfactant.This led to the activation of the cells that are localized in the alveolar walls
and on the surface. What cells are these?
A Alveolar macrophages *
B Alveolocytes type I
C Endothelial cells
D Clara cells
E Alveolocytes type II
32
Urinary system.
Kidneys. Urinary
Tract.
Electron micrograph of a kidney fragment presents an afferent arteriole with big
cells under endothelium. These cells contain secretory granules. Name this type of
cells:
A Juxtaglomerular*
B Mesangial
C Smooth muscular
D Juxtavascular
E Interstitial
A microphotography represents a fragment of cortical substance of a kidney. This
fragment contains thick spot cells and juxtaglomerular cells with big secretory
granules. What kidney structure is represented?
A Juxtaglomerular apparatus*
B Renal corpuscle
C Filtering barrier
D Prostaglandin apparatus
E Choroid glomus
Electron-microscope investigation of cortical substance of a kidney reveals some
structures lined with prismatic epithelium that normally has brush border and deep
plicae of plasmolemma in its basal part. There is a big number of mitochondrions
between these plicae. These structures belong to the following part of a nephron:
A Proximal tubule*
B Distal convoluted tubule
C Henle's loop
D Renal corpuscle
52
E Distal straight tubule
The patient with disease of kidneys which is accompanied by an ischemia of
parenchyma has the high level of arterial pressure. What conducting leading factor is
the reason of increase the AP at this patient?
A. High level of angiotensin ІІ*
B. High level of vasopressine
C. High level of renine
D. Increase of sympathetic nervous system’s activity
E. Hyperkateholemia
At the patient after a heavy trauma the shock has developed and attributes of acute
renal insufficiency (ARI) have appeared. What is conducting leading mechanism of
development ARI in this case?
A. Stasis of urine
B. Decreasing of blood pressure*
C. Increasing of blood pressure
D. Renal injury
E. Nephroptosis
Symptoms of anemia are found out in the patient with chronic glomerulonephritis.
What has caused occurrence of these symptoms?
A. Decreasing of erithropoietines’ synthesis*
B. Haematuria
C. Increasing of normal erythrocytes’ destruction
D. Haemolysis of erithricytes
E. Decreasing of synthesis of hemoglobin
At the patient of 18 years at laboratory analyses glucose presence in urine is revealed at
normal its concentration in blood plasma. The most authentic reason of it is
infringement:
A. Glomerular filtration
B. Tubular secretion
C. Glucocorticoids’ secretion
D. Secretion of insulin
E. Tubular reabsorbtion*
At electronic microscopy in renal cortex structures that lining by cylindrical epithelia
with brush border and plicaes of basal plasmalemma are defined determined.
Between plicaes there are many mitochondrias. What department of nephrons
possesses the described structures?
A. Henle’s loop
B. Proximal convoluted tubule*
C. Renal corpuscle
D. Collecting tubule
E. Capsula of nephron
At the girl of 5 years old, who was treated for cystitis, increase temperature up to 390
С is marked. On the basis of clinical and laboratory analyses the acute inflammation of
renal pelvises is diagnosed (pyelonephritis). What structural features have most likely
caused propensity of child to pyelonephritis?
A. Renal infarct
B. Undevelopment почечных лоханок
C. Short ureters*
D. Renal ectopia
E. Small diameter of ureters
Developmental anomalies of urinary system meet, on the statistical data, in 10-14 % of
newborn children. From which sources, marked below, kedney developmented?
53
A. Splanchnotome
B. Mezenchyma
C. Nephrogonotome*
D. Segmental legs
E. Allantois
During clinical inspection in the 35-years woman with disease of kidneys in urine
blood cells are found out, that is authentically connected to infringement of renal
filter. This filter will consist of what structures?
A. Endotheliocytes, podocytes
B. Three-layered basal lamina
C. Endotheliocytes, basal lamina
D. Podocytes, basal lamina
E. Endotheliocytes, basal lamina, podocytes *
In experiment increase of blood pressure is received from an animal by narrowing of
renal arteries. What function of kidneys’ cells causes this effect?
A. Podocytes
B. Juxtaglomerular cells *
C. Endotheliocytes
D. Interstitial cells
E. Cells of macula densa
At the laboratory analysis of urine of the patient it is revealed, that it has subacidic
reaction. What cells of kidneys provide this reaction of urine?
A. Cells of collecting tubules *
B. Juxtaglomerular cells
C. Juxtavascular cells
D. Cells of macula densa
E. Interstitial cells
On preparation it is well visible capillary rete, located between two arterioles (rete
mirabile). In what body it is possible to find out this picture?
A. In kidney *
B. In liver
C. In suprarenal gland
D. In spleen
E. In lungs
The patient of 46 years has addressed to the surgeon with complaints to allocation
from umbilicus a transparent liquid with unpleasant smell. At laboratory research
of this liquid it is determined, that it is urine, the diagnosis is established. Surgical
treatment is recommended. What structure it is necessary to lead carry out to the given
patient?
A. Urahus*
B. Ureter
C. Uretry
D. Umbilical vein
E. Umbilical cord
The low specific gravity of the secondary urine (1002) was found out in the sick
person. What is the most distant part of nephron where concentration of secondary
urine takes place?
A In the collecting tubules *
B In the nephron’s glomerulus
C In proximal tubule of nephron
D In ascending part of loop of Henle
E In distal tubule of nephron
54
A histological specimen of a kidney shows a part of the distal tubule going between
the afferent and efferent arterioles. The cells building the tubule wall have dense
nuclei; basal membrane is absent. Such structural formation is called:
A Macula densa *
B Juxtaglomerular cells
C Mesangial cells
D Juxtavascular cells
EA histological specimen of kidney shows a structure consisting of a glomerulus of
fenestrated capillaries and a bilayer epithelial capsule. Specify this structure:
A Renal corpuscle *
B Proximal tubule
C Distal tubule
D Henle's loop
E Receiving tube
33
Male reproductive
system. Testis.
In preparation of organ with many tubules which wall is formed by own environment
which will consist from basal, myoid and fibrous layers is submitted. On basal lamina
supporting cells and spermatogenic epithelia placed. What organ is submitted in a
preparation?
A. Epididimis
B. Testis*
C. Ductus deferens
D. Ejaculatory duct
E. Prostata
In an embryo on 2-3 week of development are found out gonocytes - predecessors of
reproductive cells. In what material these cells are differentiated?
A. In dermatomes
B. In mesenchyma
C. In germinal ectoderm
D. In yolk sac *
E. In germinal endoderm
At the man of 35 years after the received doze of an irradiation it is broken
spermatogenesis. From what cells there is a restoration of spermatogenesis?
A. Glandulocytes
B. Spermatozoa
C. Spermatogonia*
D. Sustentocytes
E. Spermatides
At the middle-aged man occurrence is observed secondary sexual attributes behind
female type. What function of testis’ cells is reduced?
A. Sustentocytes
B. Glandulocytes*
C. Spermatozoa
D. Exocrinocytes
E. Fibrocytes
In wall of yolk sac cells with the large nucleus, the raised increased maintenance
contents of glycogen and high activity of alkaline phosphotaze are found out.
Subsequently these cells with blood on vessels migrate to mesonephros. In what
cells of man's reproductive system they are differentiated?
A. Leidig’s cells
B. Supporting cells
C. Spermatogenic cells *
D. Fibroblasts
55
E. Blood cells
Activity of man's reproductive system is adjusted by nervous and endocrine systems?
What from the listed hormones adjust activity of man's reproductive system?
A. Glycocorticoids
B. Gonadotrophins
C. Thyreotroph hormon
D. Теstosterone*
E. Epinephrine
At electronic microscopy coiled seminiferous tubules’ cells of pyramidal shape have
been found out. In cytoplasm of these cells it is well advanced smooth endoplasmic
reticulum, Golgy complex and various inclusions. What cells are there?
A. Spermatogonii
B. Leidig’s cells
C. Sustentocytes *
D. Fibroblasts
E. Gonocytes
Attributes of cryptorchism are found out in the boy – undescendence testis in
scrotum. If not to perform operation, it can result in what consequences?
A. Death of the supporting cells
B. Destruction of coiled seminiferous tubules’ wall
C. Cessation of spermatogenesis*
D. Migration of testis
E. Death of the glandulocytes
At research of prostatic liquid at the patient of 25 years the insufficient quantity
amount of reproductive cells is revealed. From what cells usually provide sufficient
quantity amount for fertilisation of spermatozoa?
A. Spermatogonia *
B. Sustentocytes
C. Supporting cells
D. Leidig’s cells
E. Glandulocytes
During pubescence the cells of male sexual glands begin to produce male sex hormon
testosterone that calls forth secondary sexual characters. What cells of male sexual
glands produce this hormone?
A Leidig cells *
B Sustentocytes
C Sertoli's cells
D Supporting cells
E Spermatozoa
34
Male reproductive
tract. Accessories
glands.
At the patient with a chronic prostatitis unfertilization is observed. What reason of it?
A. Disturbance of spermatogenesis
B. Unspermatogenesis
C. Disturbance of hormonal function
D. Decreasing of mobility of spermatozoa*
E. Disturbance of immunity system
35
Female
reproductive
system. Ovaries.
In the ovary specimen colored with hematoxylin-eosin, follicle is determined where
cubic-shaped follicle epithelium cells are placed in 1-2 layers, and zona pellucida
covering is seen around ovocyte. Name this follicle:
A Primary*
B Primordial
C Secondary
D Mature
56
E Atretic
The body of unknown woman has been delivered to medicolegal examination. On
section in ovary round formation in diameter about 5 sm that contains a pigment of
yellow color is revealed. This formation will consist of what cells?
A. Interstitial cells
B. Myocytes
C. Follikular cells
D. Fibroblasts
E. Luteocytes*
At preparation of ovary are determined structures which have the big cavity. Оvocyte
in them it is surrounded with zona pellucida, zona radiata. Wall is formed by layer
of follicular cells and teka. Specify what structure of ovary has given morphological
attributes.
A. Mature follicle *
B. Premordial follicle
C. Primary follicle
D. Yellow body
E. Atretic body
In blood of the woman have found out the increased quantity amount of oestrogens.
What cells synthesize these hormones?
A. Ovocytes
B. Follicular cells of secondary follicles *
C. Follicular cells of primary follicles
D. Follicular cells of premordial follicles
E. Follicular cells and ovocytes
In the woman 50 years old it is found out cyst of ovary. From what structure it has
developed?
A. From a follicle *
B. Ovary cortex
C. Atretic body
D. White body
E. Interstitial cells
On preparation of ovary many large blood vessels is revealed. What structure of
ovary on preparation?
A .Cortex
B. Medulla *
C. Thertery follicle
D. Atretic follicle
E. Yellow body
At microscopic research of biopsy of endometrium women who suffers on unfertility,
are found out changes in its structure, caused by action of progesterone. Where this
hormone is produced?
A. In hypothalamus
B. In follicles
C. In neurohypophisis
D. In adinohypophysis
E. In yellow body *
36
Female
reproductive
system. Uterus.
Mammary gland.
The impact of oxitocine on uterus wall helps to stop uterine bleeding after labor. What
shall of this oragan react on the effect of this hormone?
A Myometrium*
B Endometrium
C Perimetrium
57
D Parametrium
E Submucous membrane
A patient underwent Caesarean section. During the operation a long incision was made
in the uterus wall and the fetus was extracted from uterus. Healing of the sutured
myometrium will proceed in the following way:
A Formation of a fibrous scar*
B Formation of smooth muscular tissue
C Formation of cross-striated muscle fibers
D Proliferation of myosatellitocytes
E Hypertrophy of smooth myocytes
After gynecologic operation, the patient of 32 years old began to complain of pain in
lower part of abdomen and profuse discharge from vagina. After inspection the
diagnosis - an inflammation of an internal shall of uterus is established. What shall
have suffered?
А. Endometrium*
В. Myometrium
С. Perimetrium
D. Parametrium
Е. Peritoneum
At biopsy of mammary gland astra-shape cells located between basal lamina and
lactocytes are found out. Named a source of development of these cells?
A. Myotome
B. Sklerotome
C. Ectoderm *
D. Dermatome
E. Endoderm
In ovarian-menstrual cycle there are changes in endometrial glands. To what type
these glands belong?
A. Simple tubular *
B. Simple tubulo-alveolar
C. Simple alveolar
D. Comound alveolar
E. Compound tubular
37
Menstrual cycle.
In a preparation of ovary near to follicles of the different stadies of development
appear atretic bodies and mature yellow body. What stage of menstrual cycle is
answered with such condition?
A. Premenstrual *
B. Menstrual
C. Postmenstrual
D. Regenerative
E. Growth of follicle
At the analysis of blood in not pregnant woman the age of 26 years finds out low
concentration of oestrogens and high of progesterone. In what stage of ovarianmenstrual cycle the analysis of blood has been made?
A. Premenstrual phase *
B. Menstrual phase
C. Postmenstrual phase
D. Desquamation
E. Proliferation
38
Control of
mastering of the
submodule 4.
58
39
Final test control
of the module 2.
59